इंदौर शाखा: IAS और MPPSC फाउंडेशन बैच-शुरुआत क्रमशः 6 मई और 13 मई   अभी कॉल करें
ध्यान दें:

प्रारंभिक परीक्षा


प्रारंभिक परीक्षा

प्रारंभिक परीक्षा, 2021: एक विश्लेषणात्मक अध्ययन

  • 24 Dec 2021
  • 179 min read

1. प्राचीन भारत के इतिहास के संदर्भ में, भवभूति, हस्तिमल्ल तथा क्षेमेश्वर क्यो प्रसिद्ध थे?

(a) जैन साधु      
(b)  नाटककार
(c) मंदिर वास्तुकार 
(d)  दार्शनिक

उत्तर:  (b)

व्याख्या: 8वीं सदी के विद्वान भवभूति संस्कृत में लिखे अपने नाटकों और कविताओं के लिये विख्यात् हैं। उनके द्वारा लिखित नाटक ‘मालती माधव प्रसिद्ध है।

  • हस्तिमल्ल होयसल राज में रहने वाले एक जैन नाटककार थे जो दिगंबर जैन कथाओं को आधार बनाकर संस्कृत में नाटक लिखते थे। उनके द्वारा कई नाटक लिखे गए जिनमें से चार नाटक प्राप्त हैं- मैथिली कल्याण, विक्रांत कौरव, अञ्जना पवनंजय और सुभद्रा (नाटिका)।
  • क्षेमेश्वर एक नाटककार थे जिन्होंने ‘चंडकौशिक नामक नाटक की रचना की थी। इस प्रकार भवभूति, हस्तिमल्ल और क्षेमेश्वर ‘नाटककार थे अत: विकल्प (b) सही है।

2. निम्नलिखित कथनों पर विचार कीजिये-

  1. 1919 के मॉन्टेग्यू-चेम्सफड़ॱर सुधारों में, 21 वर्ष से अधिक आयु की सभी महिलाओं के लिये मताधिकार की संस्तुति की गई।
  2. 1935 के गवर्नमेंट ऑफ इंडिया एक्ट में, विधानमंडल में महिलाओं के लिये आरक्षित स्थानों का प्रावधान किया गया।

उपर्युक्त कथनों में से कौन-सा/से सही है/हैं?

(a) केवल 1
(b)  केवल 2
(c) 1 और 2 दोनों  
(d)  न तो 1 और न ही 2

उत्तर: (b) 

व्याख्या: 1919 के मांटेग्यू-चेम्सफोर्ड सुधारों में 21 वर्ष से अधिक आयु की सभी महिलाओं को मताधिकार की संस्तुति नहीं की गई थी। अत: कथन 1 गलत है यद्यपि 1919 के भारत शासन अधिनियम में प्रांतीय परिषदों को महिलाओं को मताधिकार देने के नियम बनाने के लिये कहा गया था जो संपत्ति, आय और शिक्षा पर आधारित हो।

  • इस आधार पर 1919 में मद्रास शहर, 1920 में त्रावणकोर और झालवाड़, 1921 में बंबई और मद्रास प्रांत में सीमित मात्रा में महिलाओं को मताधिकार दिया गया। गोलमेज़ सम्मेलन ने मत देने के लिये महिलाओं की उम्र 21 वर्ष करने की सिफारिश की थी।
  • 1935 के अधिनियम में और अधिक महिलाओं को मताधिकार मिला लेकिन कुल मिलाकर 2.5% महिलाओं तक ही सीमित था। उनके लिये संप्रदाय के आधार पर विधान मंडल में स्थान आरक्षित किया गया। यद्यपि वे किसी भी अनारक्षित स्थान पर भी चुनाव लड़ सकती थीं। अत: कथन 2 सही है। अत: विकल्प (b) सही है।

3. भारतीय इतिहास में 8 अगस्त, 1942 के संदर्भ में, निम्नलिखित कथनों में से कौन-सा सही है?

(a) ए.आई.सी.सी. द्वारा भारत छोड़ो प्रस्ताव अंगीकार किया गया।
(b) वायसराय की एक्जेक्यूटिव काउंसिल का विस्तार अधिक संख्या में भारतीयों को सम्मिलित करने के लिये किया गया।
(c) सात प्रांतों में कांग्रेस मंत्रिमंडलों ने त्यागपत्र दिया।
(d) क्रिप्स ने प्रस्ताव रखा कि द्वितीय विश्वयुद्ध समाप्त होते ही संपूर्ण डोमिनियन स्टेटस वाले भारतीय संघ की स्थापना की जाएगी।

उत्तर: (a) 

व्याख्या: 8 अगस्त, 1942 को ए.आई.सी.सी. (All India Congress Committee) के बंबई अधिवेशन में गांधी जी ने भारत छोड़ो आंदोलन का आह्वान किया था। अत: विकल्प (a) सही है।

4. इनमें से कौन अंग्रेज़ी में अनूदित प्राचीन भारतीय धार्मिक गीतिकाव्य- ‘सॉन्ग्स फ्रॉम प्रिज़न’ से संबद्ध है?

(a) बालगंगाधर तिलक
(b)  जवाहरलाल नेहरू
(c) मोहनदास करमचंद गांधी
(d) सरोजिनी नायडू

उत्तर: (c)

व्याख्या: येरवडा जेल, पूना में गांधी जी ने भारतीय धार्मिक गीतिकाव्य का अनुवाद अंग्रेज़ी में ‘सॉन्ग्स प्रॉम प्रिज़न नाम से किया था। अत: विकल्प (c) सही है।

5. मध्यकालीन भारत के संदर्भ में, निम्नलिखित में से कौन-सा आकार की दृष्टि से आरोही क्रम में सही अनुक्रम है?

(a) परगना-सरकार-सूबा  
(b) सरकार-परगना-सूबा
(c) सूबा-सरकार-परगना  
(d)  परगना-सूबा-सरकार

उत्तर: (a)

व्याख्या: मध्यकालीन भारत में राज्य का प्रशासनिक विभाजन सूबा यानी प्रांत/प्रदेश में, सूबा का सरकार (यानी आधुनिक तहसील/अनुमंडल) और परगने का विभाजन गाँव में किया जाता था। अत: सबसे छोटी इकाई गाँव, उससे बड़ी परगना, परगना से बड़ी सरकार और सरकार से बड़ी इकाई सूबा होती थी। अत: विकल्प (a) सही है।

6. इनमें से कौन सेक्रेटरी के रूप में हिंदू फीमेल स्कूल से संबद्ध थे/थीं, जो बाद में बेथ्यून फीमेल स्कूल के नाम से जाना जाने लगा?

(a) एनी बेसेंट      
(b) देवेन्द्रनाथ टैगोर
(c) ईश्वरचंद्र विद्यासागर 
(d) सरोजिनी नायडू

उत्तर (c)

व्याख्या: बेथ्यून ने 1849 में कोलकाता में कलकत्ता महिला विद्यालय के नाम से दक्षिणा रंजन मुखर्जी के वित्तीय सहयोग से एक विद्यालय खोला जो बाद में हिंदू महिला विद्यालय और उसके बाद बेथ्यून विद्यालय के नाम से जाना गया। 1879 में इसे कॉलेज के रूप में विकसित किया गया। यह भारत का पहला वूमेन कॉलेज  बना और एशिया का पहला महिला विद्यालय। इसी विद्यालय से सेक्रेटरी के रूप में ईश्वरचंद्र विद्यासागर जुड़े हुए थे। इस प्रकार विकल्प (c) सही है।

7. औपनिवेशिक भारत के संदर्भ में, शाहनवाज़ खान, प्रेम कुमार सहगल और गुरबख्श सिंह ढिल्लों याद किये जाते हैं-

(a) स्वदेशी और बहिष्कार आंदोलन के नेता के रूप में।
(b)  1946 की अंतरिम सरकार के सदस्यों के रूप में।
(c) संविधान सभा में प्रारूप समिति के सदस्यों के रूप में।
(d) आज़ाद हिंद फौज (इंडियन नेशनल आर्मी) के अधिकारियों के रूप में।

उत्तर (d)

व्याख्या: 1945 में आज़ाद हिंद फौज के सिपाही सरदार गुरुबख्श सिंह, श्री प्रेम सहगल व शाहनवाज़ पर सरकार के प्रति निष्ठा की शपथ तोड़ने के आरोप पर लाल किले में मुकदमा चलाया गया। उनका बचाव भूलाभाई देसाई के नेतृत्व में तेबहादूर सप्रू, काटजू तथा जवाहरलाल नेहरू ने भी किया। एक संक्षिप्त सुनवाई के पश्चात् उन्हें मृत्युदंड सुनाया गया। लेकिन वायसराय लॉर्ड वेवेल ने जन-भावना को देखते हुए उनको क्षमा प्रदान कर दिया। अत: विकल्प (d) सही है।

8. भारतीय इतिहास के संदर्भ में, निम्नलिखित कथनों में से कौन-सा/से सही है/हैं?

  1. हैदराबाद राज्य से आरकोट की निज़ामत का उदय हुआ।
  2. विजयनगर साम्राज्य से मैसूर राज्य का उदय हुआ।
  3. रुहेलखंड राज्य का गठन, अहमद शाह दुर्रानी द्वारा अधिकृत राज्यक्षेत्र में से हुआ।

नीचे दिये गए कूट का प्रयोग कर सही उत्तर चुनिये-

(a)  1 और 2       
(b)  केवल 2
(c)  2 और 3       
(d)  केवल 3

उत्तर:(a)

व्याख्या: कर्नाटक जिसकी राजधानी आरकोट थी, मुगल दक्कन का एक सूबा था और इस प्रकार हैदराबाद के निज़ाम के तहत आता था। लेकिन जिस प्रकार निज़ाम ने अपने को दिल्ली की मुगल सरकार से स्वतंत्र कर लिया था, उसी प्रकार कर्नाटक के उप-सूबेदार ने दक्कन के निज़ाम से स्वयं को स्वतंत्र कर लिया। इस प्रकार कर्नाटक के नवाब सदातुल्ला खान ने अपने भतीजे दोस्त अली को अपनी गद्दी सौंपी। अत: कथन 1 सही है।

  • विजयनगर के पतन के पश्चात् मैसूर वाडयार वंश के शासन में एक स्वतंत्र राज्य के रूप में उभरा। अत: कथन 2 सही है।
  • नादिरशाह के आक्रमण के पश्चात् मुगल प्रशासन के संकट काल में अली मुहम्मद खान ने रुहेलखंड नाम से एक नए राज्य का गठन किया जो उत्तर में कुमाऊँ की पहाड़ियों तथा दक्षिण में गंगा तक विस्तृत था। आरंभ में इसकी राजधानी बरेली के आवलां में स्थित थी जो बाद में रामपुर में स्थानांतरित कर दी गई। रुहेलों का संघर्ष दिल्ली, अवध और जाट राज्य से लगातार चलता रहता था। अत: कथन 3 गलत है। अत: विकल्प (a) सही है।

9. निम्नलिखित कथनों में से कौन-सा सही है?

(a) अजंता गुफाएँ, वाघोरा नदी की घाटी में स्थित हैं।
(b) साँची स्तूप, चंबल नदी की घाटी में स्थित है।
(c) पांडू-लेणा गुफा देव मंदिर, नर्मदा नदी की घाटी में स्थित है।
(d) अमरावती स्तूप, गोदावरी नदी की घाटी में स्थित है।

उत्तर:(a) 

व्याख्या:अजंता गुफा दक्कन पठार में वाघोर नदी घाटी में स्थित है। अत: विकल्प (a) सही है।

  • साँची स्तूप, बेतवा नदी की घाटी में स्थित है।
  • पांडू-लेणा गुफा देव, मंदिर नासिक में स्थित है, जहाँ गोदावरी नदी का उद्गम स्थल है।
  • अमरावती स्तूप, कृष्णा नदी घाटी में स्थित है।

10. निम्नलिखित कथनों पर विचार कीजिये-

  1. यूनिसेफ (UNICEF) द्वारा 21 फरवरी को अंतर्राष्ट्रीय मातृभाषा दिवस घोषित किया गया।
  2. पाकिस्तान की संविधान सभा में यह मांग रखी गई कि राष्ट्रभाषाओं में बांग्ला को भी सम्मिलित किया जाए।

उपर्युक्त कथनों में से कौन-सा/से सही है/हैं?

(a) केवल 1       
(b)  केवल 2
(c)  1 और 2 दोनों  
(d)  न तो 1 और न ही 2

उत्तर:(b) 

व्याख्या 17 नवंबर, 1999 को पहली बार यूनेस्को (UNESCO) ने ‘21 फरवरी को अंतर्राष्ट्रीय मातृभाषा दिवस घोषित किया था। इसे संयुक्त राष्ट्र संघ की महासभा द्वारा अपने प्रस्ताव 56/262 द्वारा अपना लिया गया। अत: कथन 1 गलत है।

  • 7 मई, 1954 को मुस्लिम लीग के समर्थन से पाकिस्तान की संविधान सभा ने बांग्ला भाषा को राजभाषा (राष्ट्रभाषा) का दर्जा देने का प्रस्ताव पारित किया। 29 फरवरी, 1956 को पाकिस्तान के पहले संविधान में अनुच्छेद 214(1) में उर्दू के साथ बांग्ला भाषा को भी राजभाषा का दर्जा दिया गया था। अत: कथन 2 सही है। अत: विकल्प (b) सही है।

11. मुरैना के समीप स्थित चौंसठ योगिनी मंदिर के संदर्भ में, निम्नलिखित कथनों पर विचार कीजिये-

  1. यह कच्छपघात राजवंश के शासनकाल में निर्मित एक वृत्ताकार मंदिर है।
  2. यह भारत में निर्मित एकमात्र वृत्ताकार मंदिर है।
  3. इसका उद्देश्य इस क्षेत्र में वैष्णव पूजा-पद्धति को प्रोत्साहन देना था।
  4. इसके डिज़ाइन से यह लोकप्रिय धारणा बनी कि यह भारतीय संसद भवन के लिये प्रेरणा-स्रोत रहा था।

उपर्युक्त कथनों में से कौन-से सही हैं?

(a)  1 और 2       
(b)  केवल 2 और 3
(c)  1 और 4       
(d) 2, 3 और 4

उत्तर:(c)

व्याख्या: मितौली गाँव, मुरैना (मध्य प्रदेश) स्थित चौंसठ योगिनी मंदिर का निर्माण 1323 ई. में कच्छपघात राजा देवपाल ने करवाया था। अत: कथन 1 सही है।

  • हीरापुर, भुवनेश्वर स्थित चौंसठ योगिनी मंदिर भी वृत्ताकार है। अत: यह एक मात्र वृत्ताकार मंदिर नहीं है। अत: कथन 2 गलत है।
  • चौंसठ योगिनी मंदिर देवी पार्वती के मंदिर हैं जो तंत्र तथा शैव पूजा पद्धति को बढ़ावा देते हैं। अत: कथन 3 गलत है।
  • चौंसठ योगिनी मंदिर के वृत्ताकार रूप को संसद भवन के लिये प्रेरणास्रोत के रूप में देखा जाता है। अत: कथन 4 सही है। अत: विकल्प (c) सही है।

12. निम्नलिखित में से कौन-सा प्राचीन नगर अपने उन्नत जल संचयन और प्रबंधन प्रणाली के लिये सुप्रसिद्ध है, जहाँ बांधों की  शृंखला का निर्माण किया गया था और संबद्ध जलाशयों में नहर के माध्यम से जल को प्रवाहित किया जाता था?

(a) धौलावीरा       
(b) कालीबंगा
(c) राखीगढ़ी       
(d) रोपड़

उत्तर:(a) 

व्याख्या: हड़प्पा सभ्यता के एक प्रमुख नगर धौलावीरा की अनूठी विशेषताओं में से एक है। उसका उन्नत जल संचयन और प्रबंधन प्रणाली  पूरी तरह से पत्थरों से निर्मित है। यहाँ विशाल जलराशि के कुंड, बंाध, नहर और तटबंध मिले हैं। सबसे प्राचीन जलसंभरण प्रणाली का विकास यहीं हुआ था। यहाँ से स्टेडियम के अवशेष और 10 बड़े आकार के संकेताक्षर वाला अभिलेख भी मिला है। अत: विकल्प (a) सही है।

13. सत्रहवीं शताब्दी के पहले चतुर्थांश में, निम्नलिखित में से कहाँ इंग्लिश ‘ईस्ट इंडिया कंपनी’ का कारखाना/के कारखाने स्थित था/थे?

  1. भरूच
  2. चिकाकोल
  3. त्रिचिनोपोली

नीचे दिये गए कूट का प्रयोग कर सही उत्तर चुनिये-

(a) केवल 1       
(b) 1 और 2
(c) केवल 3       
(d) 2 और 3

उत्तर:(a) 

व्याख्या:सर थॉमस रो ने जहाँगीर से अनुमति लेकर अहमदाबाद, भरूच और आगरा में फैक्टरियाँ स्थापित की थीं। चिकाकोल और त्रिचनापोली में ईस्ट इंडिया कंपनी का कारखाना नहीं था। अत: विकल्प (a) सही है।

14. गुप्त वंश के पतन से लेकर आरंभिक सातवीं शताब्दी में हर्षवर्धन के उत्थान तक उत्तर भारत में निम्नलिखित में से किन राज्यों का शासन था?

1. मगध के गुप्त             
2. मालवा के परमार
3. थानेसर के पुष्यभूति  
4. कन्नौज के मौखरि
5. देवगिरि के यादव       
6. वल्लभी के मैत्रक

नीचे दिये गए कूट का प्रयोग कर सही उत्तर चुनिये-

(a) 1, 2 और 5     
(b) 1, 3, 4 और 6
(c) 2, 3 और 4     
(d) 5 और 6

उत्तर: (b)

व्याख्या: गुप्त वंश के पतन के पश्चात् तथा हर्षवर्द्धन के उत्थान तक उत्तर भारत में निम्न राजवंशों का शासन था-

  • मगध के गुप्त शासक
  • थानेसर के पूष्यभूति जिस वंश से हर्षवद्धन संबंधित था।
  • कन्नौज के मौखरि जिसके राजा गृहवर्मन से हर्ष की बहन राज्यश्री का विवाह हुआ था।
  • वल्लभी के मैत्रक वंशी ध्रुवसेन द्वितीय से हर्ष ने अपनी पुत्री का विवाह किया था।
  • मालवा के परमार और देवगिरि के यादव दक्कन में शासन किये। उनका शासनकाल 10वीं शताब्दी से 14वीं शताब्दी के मध्य था। जबकि गुप्त वंश का पतन और हर्ष का उत्थान 6-7वीं शताब्दी में हुआ था। अत: विकल्प (b) सही है।

15. पुर्तगाली लेखक नूनिज़ के अनुसार, विजयनगर साम्राज्य में महिलाएँ निम्नलिखित में से किन क्षेत्रों में निपुण थीं?

1. कुश्ती                             
2. ज्योतिषशास्त्र
3. लेखाकरण                     
4. भविष्यवाणी

नीचे दिये गए कूट का प्रयोग कर सही उत्तर चुनिये-

(a) केवल 1, 2 और 3
(b)  केवल 1, 3 और 4
(c) केवल 2 और 4  
(d) 1, 2, 3 और 4

उत्तर:(d)

व्याख्या: 1535 से 1537 के मध्य पुर्तगाली यात्री नूनिज़ ने विजयनगर की यात्रा की थी और उसने वहाँ की सामाजिक-आर्थिक दशा का विस्तृत वर्णन किया था। इस समय अच्चुत देवराय का शासन था। नूनिज़ के अनुसार विजयनगर में स्त्रियों का बहुत सम्मान था। कुछ स्त्रियाँ मल्लयोद्धा, ज्योतिषी, भविष्यवक्ता, अंगरक्षिकाएँ, सुरक्षाकर्मी, लेखाधिकारी, लिपिक एवं संगीतकार होती थीं। वे युद्ध में भी भाग लेती थीं। अत: विकल्प (d) सही है।

16. आंध्र प्रदेश में मदनपल्ली के संदर्भ में, निम्नलिखित में से कौन-सा कथन सही है?

(a) पिंगली वेंकैया ने यहाँ भारतीय राष्ट्रीय ध्वज तिरंगे का डिज़ाइन किया।
(b) पट्टाभि सीतारमैया ने यहाँ से आंध्र क्षेत्र में भारत छोड़ो आंदोलन का नेतृत्व किया।
(c) रवींद्रनाथ टैगोर ने यहाँ राष्ट्रगान का बांग्ला से अंग्रेज़ी में अनुवाद किया।
(d) मैडम ब्लावात्स्की तथा कर्नल ऑलकॉट ने सबसे पहले यहाँ थियोसोफिक़ल सोसाइटी का मुख्यालय स्थापित किया।

उत्तर:(c) 

व्याख्या: गुरुदेव रवींद्रनाथ टैगोर ने 1919 में ‘जन गण मन को बांग्ला से अंग्रेज़ी में अनुवाद मदनपल्ली (चिक्तर, आंध्र प्रदेश) के बेसेंट थियोसोफिकल कॉलेज (एनी बेसेंट द्वारा स्थापित) में किया था। इसकी धुन भी उसी समय उन्होंने बनाई। उस समय कॉलेज के प्रधानाध्यापक शिक्षाविद् जेम्स हेनरी कजिन की पत्नी मार्गरेट कजिन ने ‘जन गण मन को संगीतबद्ध किया। अत: विकल्प (c) सही है।

17. निम्नलिखित युग्मों पर विचार कीजिये-

(ऐतिहासिक स्थान)        (ख्याति का कारण)

  1. बुर्ज़होम      शैलकृत देव मंदिर
  2. चंद्रकेतुगढ़    टेराकोटा कला
  3. गणेश्वर      ताम्र कलाकृतियाँ

उपर्युक्त युग्मों में से कौन-सा/से सही सुमेलित है/हैं?

(a) केवल 1   
(b) केवल 1 और 2
(c) केवल 3     
(d) 2 और 3

उत्तर:(d)

व्याख्या: बुर्ज़होम कश्मीर का एक नवपाषाणकालीन स्थल है जहाँ से गर्तावास के साक्ष्य मिले हैं। आखेट पर आश्रित मानव इस समय मंदिर का निर्माण करना नहीं जानता था। अत: पहला युग्म गलत है।

  • उत्तरी 24 परगना ज़िला, पश्चिम बंगाल में स्थित चंद्रकेतुगढ़ का उल्लेख टॉलेमी ने ‘गंगेरीदाई’ नाम से किया है। यहाँ की खुदाई में टेराकोटा (पकी मिट्टी) की मूर्तियाँ मिली हैं। यह स्थल मौर्य, मौर्योत्तर, गुप्त-गुप्तोत्तर काल में आबाद था। यहाँ से गुप्तकालीन सोने और चांदी के सिक्के भी मिले हैं। अत: युग्म 2 सही है।
  • राजस्थान के सीकर ज़िले की नीम का थाना तहसील में स्थित गणेश्वर से ताम्र वस्तुएँ प्राप्त हुई हैं जिनमें शामिल हैं- बाणाग्र, मछली का काँटा, चूड़ी, शूलाग्र, छेनी आदि। अत: युग्म 3 सही है।
  • अत: विकल्प (d) सही है।

18. निम्नलिखित कथनों पर विचार कीजिये-

  1. इल्तुतमिश के शासनकाल में, चंगेज़ खान भगोड़े ख्वारिज़्म युवराज की खोज में सिंधु नदी तक पहुँचा था।
  2. मुहम्मद बिन तुगलक के शासनकाल में, तैमूर ने मुल्तान पर अधिकार किया था और सिंधु नदी पार की थी।
  3. विजयनगर साम्राज्य के देवराय द्वितीय के शासनकाल में, वास्को-द-गामा केरल के तट पर पहुँचा था।

उपर्युक्त कथनों में से कौन-सा/से सही है/हैं?

(a) केवल 1    
(b)  केवल 1 और 2
(c) केवल 3       
(d) 2 और 3

उत्तर:(a) 

व्याख्या: मंगोलों के महान नेता चंगेज खान ने पर्शिया (ईरान) पर आक्रमण कर उसके शासक अलाउद्दीन मुहम्मद ख्वारिज़्मशाह का साम्राज्य नष्ट कर दिया और उसे जान बचाकर कैस्पियन समुद्रतट की ओर भागने पर मजबूर कर दिया। ख्वारिज़्मशाह का बड़ा बेटा युवराज जलालुद्दीन मांगबर्नी भारत की ओर सिंधु नदी तट तक भाग कर आया, जहाँ तक मंगोलों ने उसका पीछा किया। उसने इल्तुतमिश से मंगोलों के विरुद्ध सहायता की अपील की लेकिन चंगेज खान के आक्रमण  के भय से इल्तुतमिश ने उसकी मदद करना स्वीकार नहीं किया। अत: कथन 1 सही है।

  • मुहम्मद बिन तुगलक का शासन काल 1325-1351 ई. तक था, जबकि तैमूर ने भारत पर 1398-99 के दौरान आक्रमण किया अत: कथन 2 गलत है।
  • विजयनगर के राजा देवराय द्वितीय का शासन काल 1424-1446 ई. तक था, जबकि वास्को-द-गामा 1498 ई. में भारत के कालीकट के तट पर पहुँचा था। अत: कथन 3 असत्य है। अत: विकल्प (a) सही है।

19. निम्नलिखित कथनों पर विचार कीजिये-

  1. संत फ्रांसिस ज़ेवियर, जेसुइट संघ (ऑर्डर) के संस्थापक सदस्यों में से एक थे।
  2. संत फ्रांसिस ज़ेवियर की मृत्यु गोवा में हुई तथा यहाँ उन्हें समर्पित एक गिरजाघर है।
  3. गोवा में प्रति वर्ष संत फ्रांसिस ज़ेवियर के भोज का अनुष्ठान किया जाता है।

उपर्युक्त कथनों में से कौन-से सही हैं?

(a) केवल 1 और 2  
(b)  केवल 2 और 3
(c) केवल 1 और 3  
(d) 1, 2 और 3

उत्तर:(c) 

व्याख्या: संत फ्रांसिस ज़ेवियर का जन्म स्पेन के ज़ेवियर में हुआ था। वे संत इग्नेशियस के मित्र थे और उन्होंने उनके साथ मिलकर जेसुइट संघ की स्थापना की थी। अत: कथन 1 सही है।

  1. संत फ्रांसिस ज़ेवियर की मृत्यु 3 दिसंबर, 1552 को चीन के शेगंचुआन द्वीप पर हुई। 1553 में उन्हें गोवा में दफनाया गया। अत: कथन 2 गलत है।
  2. प्रत्येक वर्ष 3 दिसंबर को गोवा में संत ज़ेवियर के भोज का अनुष्ठान किया जाता है। अत: कथन 3 सही है। अत: विकल्प (c) सही है।

20. प्राचीन भारत के इतिहास के संदर्भ में, निम्नलिखित कथनों में से कौन-सा/से सही है/हैं?

  1. मिताक्षरा ऊँची जाति की सिविल विधि थी और दायभाग निम्न जाति की सिविल विधि थी।
  2. मिताक्षरा व्यवस्था में, पुत्र अपने पिता के जीवनकाल में ही संपत्ति पर अधिकार का दावा कर सकते थे, जबकि दायभाग व्यवस्था में पिता की मृत्यु के उपरांत ही पुत्र संपत्ति पर अधिकार का दावा कर सकते थे।
  3. मिताक्षरा व्यवस्था किसी परिवार के केवल पुरुष सदस्यों के संपत्ति-संबंधी मामलों पर विचार करती है, जबकि दायभाग व्यवस्था किसी परिवार के पुरुष एवं महिला सदस्यों, दोनों के संपत्ति-संबंधी मामलों पर विचार करती है।

नीचे दिये गए कूट का प्रयोग कर सही उत्तर चुनिये-

(a) केवल 1 और 2  
(b) केवल 2
(c) केवल 1 और 3  
(d) केवल 3

उत्तर:(b) 

व्याख्या: विज्ञानेश्वर द्वारा याज्ञवल्क्य स्मृति पर लिखी गई टीका मिताक्षरा पूरे देश में (बंगाल, असम तथा उड़ीसा एवं बिहार के कुछ भागों को छोड़कर जहाँ दायभाग व्यवस्था लागू थी) संपत्ति के अधिकार के लिये कानून की सर्वमान्य पुस्तक थी। मिताक्षरा और दायभाग जाति भेद नहीं करती थी, यानी ऊँची या नीची जाति के लिये नहीं लिखी गई थी। अत: कथन 1 गलत है।

  • मिताक्षरा व्यवस्था में पिता के जीवित रहते पुत्र, पिता की संपत्ति में अधिकार का दावा कर सकता था जबकि दायभाग पिता की मृत्यु के पश्चात् ऐसे किसी दावे पर विचार करती थी। अत: कथन 2 सही है।
  • मिताक्षरा और दायभाग स्त्री-पुरुष दोनों के संपत्ति संबंधी मामलों पर विचार व्यक्त करते हैं। अत: कथन 3 गलत है। अत: विकल्प (b) सही है।

21. निम्नलिखित में से किससे किसी अर्थव्यवस्था में मुद्रा गुणक में वृद्धि होती है?

(a) बैंकों में आरक्षित नकदी निधि अनुपात में वृद्धि
(b) बैंकों के सांविधिक चलनिधि अनुपात में वृद्धि
(c) लोगों की बैंकिंग आदतों में वृद्धि
(d) देश की जनसंख्या में वृद्धि

उत्तर:(c)

व्याख्या: किसी अर्थव्यवस्था में मुद्रा गुणक उसके मौद्रिक आधार और मुद्रा आपूर्ति के संबंध को व्यक्त करता है। मुद्रा गुणक से तात्पर्य अर्थव्यवस्था में मुद्रा के स्टॉक और शक्तिशाली मुद्रा (High Powered Money) के स्टॉक के अनुपात से है।

मुद्रा गुणक = M/H

यहाँ M से तात्पर्य मुद्रा का स्टॉक और H से शक्तिशाली मुद्रा से है।

  • चूँकि, मुद्रा का स्टॉक सामान्यतया शक्तिशाली मुद्रा के मूल्य से अधिक होता है, इसलिये मुद्रा गुणक का मूल्य 1 से अधिक होता है। जनता की बैंकिंग आदतों की वृद्धि के साथ बैंक जमाओं में वृद्धि होने से बैंकों द्वारा अधिक ऋण सृजन होगा, जिससे चलन में मुद्रा के बढ़ने से मुद्रा गुणक में वृद्धि होगी। अत: विकल्प (c) सही है।

22. भारतीय अर्थव्यवस्था के संदर्भ में, मांग-प्रेरित मुद्रास्फीति या उसमें वृद्धि निम्नलिखित किन कारणों से होती है?

  1. विस्तारकारी नीतियाँ
  2. राजकोषीय प्रोत्साहन
  3. मुद्रास्फीति सूचकांकन मज़दूरी (इनफ्लेशन-इंडेक्स़िग वेजेज़)
  4. उच्च क्रय शक्ति
  5. बढ़ती ब्याज दर

नीचे दिये गए कूट का प्रयोग कर सही उत्तर चुनिये-

(a) केवल 1, 2 और 4         
(b) केवल 3, 4 और 5
(c) केवल 1, 2, 3 और 5
(d) 1, 2, 3, 4 और 5

उत्तर:(a)

व्याख्या: जब अर्थव्यवस्था में साधन लागत एकसमान रहती है, किंतु वस्तुओं और सेवाओं की आपूर्ति की अपेक्षा उसकी मांग अधिक हो जाती है तो उसे ‘मांग-प्रेरित मुद्रास्फीति कहते हैं।

  • मांग-प्रेरित मुद्रास्फीति के कारणों में, लोगों की आय बढ़ने से उपजी उच्च क्रय शक्ति, सरकारी व्यय में तीव्र वृद्धि, बैंकों द्वारा अधिक मात्रा में ऋण देना तथा जनसंख्या वृद्धि एवं नगरीकरण आदि करने के परिणामस्वरूप मांग बढ़ना शामिल हैं।
  • इसे विस्तारपूर्वक समझने का प्रयास करें तो, सरकार अर्थव्यवस्था में सकल मांग बढ़ाने और आर्थिक विकास को बढ़ावा देने के लिये राजकोषीय प्रोत्साहन के अंतर्गत ‘कर प्रोत्साहन छूट’ जैसी पहल कर सकती है।
  • इस दृष्टिकोण के पीछे यह तर्क होता है कि यदि लोगों के कर अदायगी के भार को कम कर दिया जाए तो उनके पास व्यय करने या निवेश करने के लिये अधिक धन रहता है जो उच्च मांग को बढ़ावा देता है।
  • परिणामस्वरूप उत्पादन की मांग बढ़ने से रोज़गार सृजन होता है, जिससे बेरोज़गारी में कमी आती है।
  • इसके अलावा, सरकार अपने व्यय में वृद्धि कर आर्थिक विस्तार कर सकती है, जैसे- आधारभूत संरचना में निवेश के ज़रिये रोज़गार को बढ़ावा देकर मांग और विकास में वृद्धि की जा सकती है। अत: विकल्प (a) सही है।

23. भारत के संदर्भ में, निम्नलिखित कथनों पर विचार कीजिये-

  1. खुदरा निवेशक डीमैट खातों के माध्यम से प्राथमिक बाज़ार में ‘राजकोष बिल (ट्रेज़री बिल)’ और ‘भारत सरकार के ऋण बॉण्ड में निवेश कर सकते हैं।
  2. ‘बातचीत से तय लेन-देन प्रणाली-ऑर्डर मिलान (नेगोशिएटेड डीलिंग सिस्टम-ऑर्डर मैचिंग)’ भारतीय रिज़र्व बैंक का सरकारी प्रतिभूति व्यापारिक मंच है।
  3. ‘सेंट्रल डिपोज़िटरी सर्विसेज़ लिमिटेड का भारतीय रिज़र्व बैंक एवं बंबई स्टॉक एक्सचेंज द्वारा संयुक्त रूप से प्रवर्तन किया जाता है।

उपर्युक्त कथनों में से कौन-सा/से सही है/हैं?

(a) केवल 1   
(b)  केवल 1 और 2
(c) केवल 3       
(d) 2 और 3

उत्तर:(b) 

व्याख्या: फरवरी 2021 में, भारतीय रिज़र्व बैंक (आरबीआई) ने खुदरा निवेशकों को आरबीआई के साथ गिल्ट खाते खोलकर सीधे सरकारी बॉण्ड खरीदने की अनुमति दी। आरबीआई ने खुदरा निवेशकों को आरबीआई के माध्यम से सरकारी प्रतिभूति बाज़ार में पहुँच प्रदान की है। अत: कथन (1) सही है।

  • इसके तहत खुदरा निवेशक गैर-प्रतिस्पर्धी बोलियों के लिये स्टॉक एक्सचेंजों पर खुद को पंजीकृत करके सरकारी बॉण्ड खरीद सकते हैं। खुदरा निवेशकों के लिये सरकारी बॉण्ड खरीदने का एक अन्य तरीका सरकारी प्रतिभूतियाँ म्यूचुअल फंड हैं। बातचीत से तय लेनदेन प्रणाली-ऑर्डर मिलान [नेगोशिएटेड डीलिंग सिस्टम-ऑर्डर मैचिंग (एनडीएस-ओएम)] आरबीआई के स्वामित्व वाला सरकारी प्रतिभूतियों का व्यापारिक मंच है। इसकी सदस्यता बैंकों, प्राथमिक डीलरों, बीमा कंपनियों, म्यूचुअल फंड आदि जैसी संस्थाओं के लिये खुली है। अत: कथन (2)  सही है।
  • सेंट्रल डिपॉजिटरी सर्विसेज लिमिटेड (सीडीएसएल) को बंबई स्टॉक एक्सचेंज (बीएसई) द्वारा प्रवर्तित किया गया था। अत: कथन (3) सही नहीं है। अत: विकल्प (b) सही है।

24. ‘वाटरक्रेडिट’ के संदर्भ में, निम्नलिखित कथनों पर विचार कीजिये-

  1. यह जल एवं स्वच्छता क्षेत्र में कार्य के लिये सूक्ष्म वित्त साधनों (माइक्रोफाइनेंस टूल्स) को लागू करता है।
  2. यह एक वैश्विक पहल है जिसे विश्व स्वास्थ्य संगठन और विश्व बैंक के तत्त्वावधान में प्रारंभ किया गया है।
  3. इसका उद्देश्य निर्धन व्यक्तियों को सहायिकी के बिना अपनी जल-संबंधी आवश्यकताओं को पूरा करने के लिये समर्थ बनाना है।

उपर्युक्त कथनों में से कौन-सेे सही हैं?

(a) केवल 1 और 2
(b)  केवल 2 और 3
(c) केवल 1 और 3  
(d) 1, 2 और 3

उत्तर:(c) 

व्याख्या: अमेरिका स्थित एनजीओ Water.org अपनी वाटरक्रेडिट पहल के माध्यम से विभिन्न देशों में सूक्ष्म वित्तीय संस्थानों और गैर-सरकारी संगठनों को जल आपूर्ति और स्वच्छता के लिये सूक्ष्म वित्तीय साधनों (Microfinance Tools) को लागू करता है। इसका उद्देश्य निर्धन व्यक्तियों को सहायिकी (Subsidy) के बिना अपनी जल संबंधी आवश्यकताओं को पूरा करने के लिये समर्थ बनाना है। यह ज़रूरतमंद परिवारों को पानी और स्वच्छता के लिये किफायती वित्तपोषण प्रदान करने के लिये चयनित संस्थानों के साथ साझेदारी करता है। अत: कथन (1) और (3) सही हैं।

  • Water.org एक वैश्विक गैर-लाभकारी संगठन है जो दुनिया में पानी और स्वच्छता लाने के लिये  काम कर रहा है। इसने सुरक्षित पानी और स्वच्छता के लिये किफायती वित्तपोषण की बाधा को दूर करने के लिये ‘वाटरक्रेडिट कार्यक्रम की पहल शुरू की। अत: कथन (2) सही नहीं है। अत: विकल्प (c) सही है।

25. भारत में, ‘अंतिम उधारदाता (लेंडर ऑफ लास्ट रिसॉर्ट)’ के रूप में केंद्रीय बैंक के कार्य में सामान्यत: निम्नलिखित में से क्या सम्मिलित है/हैं?

  1. अन्य स्रोतों से ऋण प्राप्ति में विफल होने पर व्यापार एवं उद्योग निकायों को ऋण प्रदान करना
  2. अस्थायी संकट के समय बैंकों के लिये चलनिधि उपलब्ध कराना
  3. बजटीय घाटों के वित्तीयन के लिये सरकारों को ऋण देना

नीचे दिये गए कूट का प्रयोग कर सही उत्तर चुनिये-

(a)  1 और 2                     
(b)  केवल 2
(c)  2 और 3   
(d)  केवल 3

उत्तर:(b)

व्याख्या: भारतीय रिज़र्व बैंक (आरबीआई) देश की बैंकिंग और वित्तीय प्रणाली के कार्यान्वयन हेतु बैंकिंग परिचालन के विस्तृत मानदंड निर्धारित करता है। इसे बैंकाें का बैंक कहा जाता है जो समस्त वित्तीय प्रणाली और निजी बैंकों में पर्याप्त नकदी की उपलब्धता दैनिक आधार पर करता है तथा सभी बैंकों के लिये अंतिम उधारदाता (लेंडर ऑफ लास्ट रिसॉर्ट) की भूमिका निभाता है। अत: कथन (2) सही है।

  • आरबीआई, व्यापार एवं उद्योग निकाय को अन्य स्रोतों से ऋण प्राप्ति  की विफलता पर तथा सरकार को बजटीय घाटों की पूर्ति के लिये ऋण प्रदान नहीं करता है। अत: कथन 1 और 3 सही नहीं हैं। अत: विकल्प (b) सही है।

26. निम्नलिखित में से किसके अंगीकरण को प्रोत्साहित करने के लिये ‘R2 व्यवहार संहिता (R2 कोड ऑफ प्रैक्टिसेज़)’ साधन उपलब्ध करती है?

(a) इलेक्ट्रॉनिकी पुनर्चक्रण उद्योग में पर्यावरणीय दृष्टि से विश्वसनीय व्यवहार
(b) रामसर कन्वेंशन के अंतर्गत ‘अंतर्राष्ट्रीय महत्त्व की आर्द्र भूमि का पारिस्थितिक प्रबंधन
(c) निम्नीकृत भूमि पर कृषि फसलों की खेती का संधारणीय व्यवहार
(d) प्राकृतिक संसाधनों के दोहन में ‘पर्यावरणीय प्रभाव आकलन

उत्तर:(a) 

व्याख्या: R2 व्यवहार संहिता में R2 का अर्थ रिस्पांसिबल रिसाइक्ल़िग है। यह सस्टेनेबल इलेक्ट्रॉनिक्स रिसाइक्ल़िग इंटरनेशनल (SERI)  द्वारा विशेष रूप से इलेक्ट्रॉनिकी पुनर्चक्रण उद्योगों के लिये बनाया गया एक मानक है।

  • इलेक्ट्रॉनिकी पुनर्चक्रण कंपनी जो इस व्यवहार संहिता के अनुसार R2 प्रमाणित है, अपने ऑपरेटिंग सिस्टम और प्रक्रियाओं में सुधार और प्रमाणन द्वारा प्रदान की गई स्थिति के माध्यम से उच्च लाभ सीमा और अतिरिक्त बाज़ार हिस्सेदारी प्राप्त करने से लाभान्वित होगी।
  • एक R2 प्रमाणित कंपनी अपने ग्राहकों को आश्वस्त करने में सक्षम होगी कि वह पर्यावरण, कार्यकर्त्ता, सार्वजनिक स्वास्थ्य और डेटा सुरक्षा की रक्षा के लिये अपनी सुविधा पर उचित उपाय करती है। अत: विकल्प (a) सही है।

27. ताम्र प्रगलन संयंत्रों के बारे में चिन्ता का कारण क्या है?

  1. वे पर्यावरण में कार्बन मोनोक्साइड को घातक मात्राओं में निर्मुक्त कर सकते हैं।
  2. ताम्रमल (कॉपर स्लैग) पर्यावरण में कुछ भारी धातुओं के निक्षालन (लीचिंग) का कारण बन सकता है।
  3. वे सल्फर डाइऑक्साइड को एक प्रदूषक के रूप में निर्मुक्त कर सकते हैं।

नीचे दिये गए कूट का प्रयोग कर सही उत्तर चुनिये-

(a) केवल 1 और 2  
(b)  केवल 2 और 3
(c) केवल 1 और 3  
(d) 1, 2 और 3

उत्तर:(b) 

व्याख्या: ताम्र (तांबा) प्रगलन संयंत्र द्वारा ताम्र सांद्र से तांबा को अलग किया जाता है जो कई सल्फाइड ऑक्सीकरण चरणों के माध्यम से होता है। इस प्रक्रिया में सल्फर ऑक्साइड प्रदूषक के रूप में निर्मुक्त होता है। इस गलाने की प्रक्रिया में लगातार काम करने वाली फ्लैश स्मेल्ट़िग फर्नेस (एफएसएफ) और बैचों में संचालित कई पियर्स-स्मिथ कन्वर्टर्स शामिल हैं। इसके अंतर्गत ताम्रमल (कॉपर स्लैग) प्रगलन प्रक्रिया में तांबे के निष्कर्षण से बना उप-उत्पाद है। गलाने के दौरान अशुद्धियाँ स्लैग बन जाती हैं जो पिघली हुई धातु पर तैरती हैं। यह पर्यावरण में भारी धातुओं के निक्षालन का कारण बन सकता है। इन भारी धातुओं में, विशेष रूप से आर्सेनिक, कैडमियम और लेड की उच्च सांद्रता हो सकती है। अत: विकल्प (b) सही है।

28. भट्टी तेल (फर्नेस ऑयल) के संदर्भ में, निम्नलिखित कथनों पर विचार कीजिये-

  1. यह तेल परिष्करणियों (रिफाइनरी) का एक उत्पाद है।
  2. कुछ उद्योग इसका उपयोग ऊर्जा (पावर) उत्पादन के लिये करते हैं।
  3. इसके उपयोग से पर्यावरण में गंधक का उत्सर्जन होता है।

उपर्युक्त कथनों में से कौन-से सही हैं?

(a) केवल 1 और 2  
(b)  केवल 2 और 3
(c) केवल 1 और 3  
(d) 1, 2 और 3

उत्तर:(d)

व्याख्या: भट्टी तेल( फर्नेस ऑयल) तेल परिष्करणियों (रिफाइनरी) का एक उत्पाद है। उद्योगों में ऊर्जा के स्रोत के रूप में इन ईंधनों का उपयोग किया जाता है। बड़े जेनरेटरों और स्टील उद्योग में इसका बड़े पैमाने पर इस्तेमाल किया जाता है। जब इन्हें जलाया जाता है तो पर्यावरण में भारी मात्रा में सल्फर (गंधक) का उत्सर्जन होता है, जो हवा को ज़हरीला बनाता है। फर्नेस तेल में सल्फर का स्तर 20000 पीपीएम तक होता है। अत: विकल्प (d) सही है।

29. ब्लू कार्बन क्या है?

(a) महासागरों और तटीय पारिस्थितिक तंत्रों द्वारा प्रगृहीत कार्बन
(b) वन जैव मात्रा (बायोमास) और कृषि मृदा में प्रच्छादित कार्बन
(c) पेट्रोलियम और प्राकृतिक गैस में अंतर्विष्ट कार्बन
(d) वायुमंडल में विद्यमान कार्बन

उत्तर:(a) 

व्याख्या: समुद्री एवं तटीय पारिस्थितिकी तंत्र द्वारा अवशोषित किये जाने वाले कार्बन को नीले कार्बन अथवा ब्लू कार्बन की संज्ञा दी जाती है। यह अवशोषण जीवभार और अवसाद के रूप में मैंग्रोव, दलदलीय क्षेत्रों, समुद्री घास तथा शैवालों द्वारा किया जाता है। महासागरों की कार्बन अवशोषण क्षमता भूमि पर स्थित पारितंत्र के मुकाबले पाँच गुना अधिक होती है। महासागर एक महत्त्वपूर्ण कार्बन सिंक (ब्लू कार्बन) के रूप में है और जलवायु परिवर्तन को कम करने में यह मददगार हो सकते हैं। अत: विकल्प  (a) सही है।

30. प्रकृति में, निम्नलिखित में से किस जीव का/किन जीवों के मृदाविहीन सतह पर जीवित पाए जाने की सर्वाधिक संभावना है?

1. फर्न                                
2. लाइकेन
3. मॉस           
4. छत्रक (मशरूम)

नीचे दिये गए कूट का प्रयोग कर सही उत्तर चुनिये-

(a) केवल 1 और 4
(b)  केवल 2
(c)  2 और 3       
(d) 1, 3 और 4

उत्तर:(c) 

व्याख्या: लाइकेन समेत मॉस ऐसे सजीव हैं, जो चटेानों पर उगते हैं। इस प्रकार इनकी मृदाविहीन सतह पर जीवित पाए जाने की संभावना हैं। इनका पारिस्थितिक दृष्टि से अत्यधिक महत्त्व है। इन्होनें चटेानों को अपघटित किया और अन्य उच्च कोटि के पौधों को उगने के अनुरूप बनाया।

  • चूँकि, मॉस मिट्टी पर एक सघन परत बना देते हैं, इसलिये वर्षा की बौछारें मिट्टी को अधिक  नुकसान नहीं पहुँचा पातीं और इस प्रकार यह मृदा अपक्षरण को रोकते हैं।
  • वहीं लाइकेन एक मिश्रित जीव है, जिसमें दो अलग-अलग जीवों, एक कवक और एक शैवाल के बीच पारस्परिक कल्याणकारी सहजीविता होती है। लाइकेन प्रदूषित क्षेत्रों में नहीं उगता है अर्थात् यह प्रदूषण के अच्छे संकेतक होते हैं। मशरूम मिट्टी में, लटॅे तथा वृक्षों के ठूँठों पर और सजीव पादपों के अंदर परजीवी के रूप में उगते हैं। अत: विकल्प (c) सही है।

31. निम्नलिखित कथनों पर विचार कीजिये-

  1. केंद्र सरकार द्वारा रिज़र्व बैंक (आर.बी.आई.) के गवर्नर की नियुक्ति की जाती है।
  2. भारतीय संविधान के कतिपय प्रावधान केंद्र सरकार को जनहित में आर.बी.आई. को निर्देश देने का अधिकार देते हैं।
  3. आर.बी.आई. का गर्वनर अपना अधिकार (पावर) आर.बी.आई. अधिनियम से प्राप्त करता है।

उपर्युक्त कथनों में से कौन-से सही हैं?

(a) केवल 1 और 2            
(b)  केवल 2 और 3
(c) केवल 1 और 3  
(d) 1, 2 और 3

उत्तर:(c)

व्याख्या: भारतीय रिज़र्व बैंक (आरबीआई) का केंद्रीय निदेशक बोर्ड इसके कारोबार का पर्यवेक्षण करता है। भारतीय रिज़र्व बैंक अधिनियम, 1934 की धारा 8 के अनुसार केंद्र सरकार द्वारा इस निदेशक बोर्ड में एक गवर्नर और अधिकतम 4 उप-गवर्नर नियुक्त किये जाते हैं। अत: कथन 1 सही है।

  • भारतीय रिज़र्व बैंक अधिनियम, 1934 की धारा 7 के अनुसार केंद्र सरकार बैंक के गवर्नर के परामर्श से समय-समय पर बैंक को जनहित में आवश्यक निर्देश दे सकती है। अत: कथन  2 सही नहीं है।
  • आरबीआई गवर्नर भारतीय रिज़र्व बैंक अधिनियम, 1934 से अपना अधिकार प्राप्त करता है। इसकी धारा 7 के अनुसार केंद्रीय निदेशक बोर्ड द्वारा बनाए गए विनियमों में गवर्नर और उनकी अनुपस्थिति में उनके द्वारा इस निमित्त नामित उप-गवर्नर के पास बैंक के मामलों और व्यवसाय के सामान्य अधीक्षण और निर्देशन की शक्तियाँ होती हैं। अत: कथन सही है। अत: विकल्प (c) सही है।

32. भारत में नियोजित अनियत मज़दूरों के संदर्भ में, निम्नलिखित कथनों पर विचार कीजिये-

  1. सभी अनियत मज़दूर, कर्मचारी भविष्य निधि सुरक्षा के हकदार हैं।
  2. सभी अनियत मज़दूर नियमित कार्य-समय एवं समयोपरि भुगतान के हकदार हैं।
  3. सरकार अधिसूचना के द्वारा यह विनिर्दिष्ट कर सकती है कि कोई प्रतिष्ठान या उद्योग केवल अपने बैंक खातों के माध्यम से मज़दूरी का भुगतान करेगा।

उपर्युक्त कथनों में से कौन-से सही हैं?

(a) केवल 1 और 2            
(b)  केवल 2 और 3
(c) केवल 1 और 3  
(d) 1, 2 और 3

उत्तर:(d)

व्याख्या: जनवरी 2020 में सर्वोच्च न्यायालय ने ‘पवन हंस लिमिटेड बनाम एविएशन कर्मचारी संगठन वाद में निर्णय दिया था कि एक नियोक्ता संविदा और स्थायी कर्मचारियों के बीच अंतर नहीं कर सकता है। कर्मचारी भविष्य निधि और प्रकीर्ण उपबंध अधिनियम, 1952 के तहत अनियत कर्मचारी भी सामाजिक सुरक्षा लाभों के हकदार हैं। उल्लेखनीय है कि इस अधिनियम को सामाजिक सुरक्षा संहिता, 2020 में अन्य 8 कानूनों के साथ समाहित कर दिया गया है। अत: कथन 1 सही है।

  • मज़दूरी संहिता, 2019 की धारा 13 और 14 में सामान्य कार्यदिवस में कार्य के घंटे का निर्धारण और समयोपरि (Overtime) भुगतान का प्रावधान है। अत: कथन 2 सही है।
  • मज़दूरी संदाय अधिनियम, 1936 में मज़दूरी संदाय (संशोधन) अधिनियम, 2017 के माध्यम से संशोधन किया गया था। जिसके तहत नियोक्ता को कर्मचारियों को सिक्कों या करंसी नोट्स में या चेक द्वारा या कर्मचारियों के बैंक खाते में जमा करके वेतन भुगतान की अनुमति दी गई थी। इसके तहत चेक द्वारा या कर्मचारियों के बैंक खाते में जमा करके वेतन भुगतान करने की स्थिति में कर्मचारी की लिखित अनुमति लेने संबंधी शर्त हटा दी गई। हालाँकि केंद्र या राज्य सरकार कुछ विशिष्ट औद्योगिक या अन्य स्थापन्न को यह निर्देश दे सकती है कि उनके नियोक्ता को अपने कर्मचारियों को केवल चेक द्वारा, या कर्मचारी के बैंक खाते में जमा करने के माध्यम से ही वेतन भुगतान करना होगा। अत: कथन 3 सही है। अत: विकल्प (d) सही है।

33. आर्थिक मंदी के समय, निम्नलिखित में से कौन-सा कदम उठाए जाने की सर्वाधिक संभावना होती है?

(a) कर की दरों में कटौती के साथ-साथ ब्याज दर में वृद्धि करना
(b) सार्वजनिक परियोजनाओं पर व्यय में वृद्धि करना
(c) कर की दरों में वृद्धि के साथ-साथ ब्याज दर में कमी करना
(d) सार्वजनिक परियोजनाओं पर व्यय में कमी करना

उत्तर:(b) 

व्याख्या: जब किसी देश की अर्थव्यवस्था लगातार दो तिमाही तक ऋणात्मक विकास दर अर्जित करती है तो इसे ‘मंदी कहा जाता है। मंदी का मूल कारण अर्थव्यवस्था में मांग का अभाव होता है। मंदी में अर्थव्यवस्था में निवेश तथा उत्पादन में गिरावट होती है। मंदी के प्रभाव को दूर करने के लिये सरकार के साथ केंद्रीय बैंक को क्रमश: विस्तारवादी राजकोषीय और मौद्रिक नीति का पालन करना चाहिये।  

  • कर की दरों में कटौती के साथ ब्याज दर में वृद्धि से अर्थव्यवस्था में ऋण महँगा हो जाएगा, जिससे निवेश हतोत्साहित होगा, जो मंदी के समय वांछनीय नहीं है। अत: विकल्प (a) सही नहीं होगा।
  • अर्थव्यवस्था को प्रोत्साहित करने के लिये सार्वजनिक परियोजनाओं पर व्यय में वृद्धि एक उचित उपकरण है क्योंकि यह पुनर्निवेश को बढ़ावा देती है। जिससे जीडीपी और अर्थव्यवस्था में आय में वृद्धि होती है। इससे मांग में वृद्धि से मंदी से उबरने में मदद मिलती है। अत: विकल्प (b) सही है।
  • कर दरों में वृद्धि से निवेशकों की आय में कमी होगी जो अर्थव्यवस्था में निवेश को हतोत्साहित करेगी। इसलिये मंदी के समय कर की दर में वृद्धि वांछनीय नहीं है। अत: विकल्प (c) सही नहीं होगा।
  • सार्वजनिक परियोजनाओं पर खर्च में कमी मंदी के समय वांछनीय नहीं है क्योंकि इससे सरकारी खर्च में कमी होगी। अत: विकल्प (d) सही नहीं होगा।

34. निम्नलिखित कथनों पर विचार कीजिये-

अन्य बातें अपरिवर्तित रहने पर भी किसी वस्तु के लिये बाज़ार मांग बढ़ सकती है, यदि

  1. इसकी स्थानापन्न वस्तु की कीमत में वृद्धि हो।
  2. इसकी पूरक वस्तु की कीमत में वृद्धि हो।
  3. वस्तु घटिया किस्म की है और उपभोक्ताओं की आय में वृद्धि होती है।
  4. इसकी कीमत घटती है।

उपर्युक्त कथनों में से कौन-से सही हैं?

(a) केवल 1 और 4             
(b) 2, 3 और 4
(c) 1, 3 और 4      
(d) 1, 2 और 3

उत्तर:(a) 

व्याख्या: अन्य कारकों के अपरिवर्तित रहने पर उपभोक्ता की किसी वस्तु के लिये मांग और वस्तु की कीमत के बीच संबंध साधारणता नकारात्मक होता है। दूसरे शब्दों में, वस्तु की मात्रा जो उपभोक्ता का इष्टतम चयन होगा, वह वस्तु की कीमत गिरने से संभावित रूप से बढ़ सकती है और वस्तु की कीमत में वृद्धि के साथ घट सकती है। अत: कथन (4) सही है।

  • उपभोक्ता की आय में वृद्धि होने के साथ वस्तु की प्रकृति के आधार पर किसी वस्तु की मांग बढ़ या घट सकती है। अधिकांश वस्तुओं की मांग उपभोक्ता की आय बढ़ने पर बढ़ती है और उपभोक्ता की आय घटने पर घट जाती है। ऐसी वस्तुएँ ‘सामान्य वस्तु कहलाती हैं। हालाँकि, कुछ ऐसी वस्तुएँ भी होती हैं जिनकी मांग उपभोक्ता की आय के विपरीत दिशा में चलती है। ऐसी वस्तुओं को ‘निम्नस्तरीय (घटिया) वस्तु कहा जाता है। जैसे-जैसे उपभोक्ता की आय बढ़ती है, निम्नस्तरीय वस्तु की मांग घटती जाती है और आय घटने पर निम्नस्तरीय वस्तु की मांग बढ़ती जाती है। निम्नस्तरीय वस्तुओं में निम्न गुणवत्ता वाले खाद्य पदार्थ जैसे मोटे अनाज शामिल हैं। अत: कथन 3 सही नहीं है।
  • एक वस्तु की मात्रा जिसका चयन उपभोक्ता करता है, किसी संबंधित वस्तु की मूल्य वृद्धि के साथ बढ़ या घट सकती है, यह इस बात पर निर्भर करता है कि दोनों वस्तुएँ एक दूसरे की ‘स्थानापन्न  (Substitute)’ या ‘पूरक (Complement)’ हैं या नहीं। जिन वस्तुओं का साथ-साथ उपयोग किया जाता है उन्हें ‘पूरक वस्तुएँ कहा जाता है। जैसे- चाय और चीनी, जूते और जुराब, कलम और स्याही इत्यादि। क्योंकि चाय और चीनी का एक साथ उपयोग में लाए जाते हैं, संभव है कि चीनी की कीमत में वृद्धि से चाय के लिये मांग घटाएगी तथा चीनी कीमत में कमी चाय की मांग में वृद्धि करेगी। पूरक वस्तुओं के विपरीत स्थानापन्न का एक साथ सेवन नहीं किया जाता है जैसे- चाय और कॉफी। वास्तव में चाय और कॉफी एक दूसरे के विकल्प हैं। चूँकि चाय कॉफी का विकल्प है, अगर कॉफी की कीमत बढ़ती है, तो उपभोक्ता चाय की ओर रुख कर सकते हैं, और इसलिये, चाय की मांग बढ़ने की संभावना है। दूसरी ओर, यदि कॉफी की कीमत घटती है, तो चाय की मांग कम होने की संभावना है। किसी वस्तु की मांग सामान्यत: उसके स्थानापन्नों की कीमत की दिशा में चलती है। अत: कथन 1 सही है, किंतु कथन 2 सही नहीं है। अत: विकल्प (a) सही होगा।

35. भारत में ‘शहरी सहकारी बैंकों’ के संदर्भ में, निम्नलिखित कथनों पर विचार कीजिये-

  1. राज्य सरकारों द्वारा स्थापित स्थानीय मंडलों द्वारा उनका पर्यवेक्षण एवं विनियमन किया जाता है।
  2. वे इक्विटी शेयर और अधिमान शेयर जारी कर सकते हैं।
  3. उन्हें वर्ष 1966 में एक संशोधन के द्वारा बैंककारी विनियमन अधिनियम, 1949 के कार्य-क्षेत्र में लाया गया था।

उपर्युक्त कथनों में से कौन-सा/से सही है/हैं?

(a) केवल 1                       
(b)  केवल 2 और 3
(c) केवल 1 और 3  
(d) 1, 2 और 3

उत्तर:(b) 

व्याख्या: बैंकिंग विनियमन (संशोधन) अधिनियम, 2020 प्राथमिक (शहरी) सहकारी बैंकों (यूसीबी) को आरबीआई की अनुमति से इक्विटी शेयर और अधिमान शेयर के माध्यम से पूंजी जुटाने की अनुमति प्रदान करता है। अत: कथन 2 सही है।

शहरी सहकारी बैंकों की व्यावसायिकता के बारे में चिंताओं ने बेहतर विनियमन की ओर बल दिया। बड़े सहकारी बैंकों, जिनकी चुकता शेयर पूंजी और कोष ` 1 लाख है, को 1 मार्च, 1966 से बैंकिंग विनियमन अधिनियम, 1949 ( मन आरबीआई और राज्य के रजिस्ट्रार जनरल ऑफ कॉपरेटिव सोसायटीज द्वारा किया जाता है। अत: कथन 1 सही नहीं है। अत: विकल्प (b) सही उत्तर होगा।

36. भारतीय सरकारी बॉण्ड प्रतिफल निम्नलिखित में से किससे/किनसे प्रभावित होता है/होते हैं?

  1. यूनाइटेड स्टेट्स फेडरल रिज़र्व की कार्रवाई
  2. भारतीय रिज़र्व बैंक की कार्रवाई
  3. मुद्रास्फीति एवं अल्पावधि ब्याज दर

नीचे दिये गए कूट का प्रयोग कर सही उत्तर चुनिये-

(a) केवल 1 और 2
(b) केवल 2
(c) केवल 3
(d) 1, 2 और 3

उत्तर:(d)

व्याख्या: बॉण्ड प्रतिफल एक निवेशक को उस बॉण्ड पर मिलने वाला प्रतिफल (रिटर्न) है। यह प्रतिफल बॉण्ड की कीमत पर निर्भर करता है जो इसकी मांग से प्रभावित होता है। प्रतिफल को प्रभावित करने वाले प्रमुख कारक भारतीय रिज़र्व बैंक की मौद्रिक नीति, सरकार की वित्तीय स्थिति, सरकार का ऋण कार्यक्रम, वैश्विक बाज़ार की स्थिति और मुद्रास्फीति की दर हैं।

  • यूनाइटेड स्टेट्स फेडरल रिज़र्व की कार्रवाइयाँ भी भारतीय सरकारी बॉण्ड के प्रतिफल को प्रभावित कर सकती हैं। उदाहरण के लिये यदि फेडरल रिज़र्व ब्याज दर में वृद्धि करता है तो निवेशक सरकारी बॉण्ड का विक्रय करेंगे, जिसके फलस्वरूप बॉण्ड के मूल्य में कमी होगी और बॉण्ड प्रतिफल में वृद्धि होगी। अत: कथन 1 सही है।
  • रिज़र्व बैंक अपने विभिन्न मुद्रास्फीति प्रबंधन उपकरणों के माध्यम से अर्थव्यवस्था में उपलब्ध तरलता और कोष की लागत का निर्धारण करता है। कोष की लागत बाज़ार में सरकारी बॉण्ड की मांग को सीधे प्रभावित करेगी और इस तरह इस पर प्रतिफल को प्रभावित करेगी। अत: कथन 2 सही है।
  • मुद्रास्फीति और अल्पकालिक ब्याज दरें अर्थव्यवस्था में लोगों की क्रय क्षमता निर्धारित करती हैं। इसलिये, इसका सरकारी बॉण्ड की मांग और कीमत पर भी असर पड़ता है जिससे बॉण्ड प्रतिफल प्रभावित होता है। अत: कथन (3) सही है। अत: विकल्प (d) सही होगा।

37. निम्नलिखित पर विचार कीजिये-

  1. विदेशी मुद्रा संपरिवर्तनीय बॉण्ड
  2. कुछ शर्तों के साथ विदेशी संस्थागत निवेश
  3. वैश्विक निक्षेपागार (डिपॉजिटरी) प्राप्तियाँ
  4. अनिवासी विदेशी जमा

उपर्युक्त में से किसे/किन्हें विदेशी प्रत्यक्ष निवेश में सम्मिलित किया जा सकता है/किये जा सकते हैं?

(a)  1, 2 और 3
(b)  केवल 3
(c)  2 और 4       
(d) 1 और 4

उत्तर: (a)

व्याख्या: समेकित एफडीआई नीति, 2020 के अनुसार विदेशी मुद्रा संपरिवर्तनीय बॉण्ड (Foreign Currency Convertible Bonds) विदेशी प्रत्यक्ष निवेश में शामिल किये जाते हैं। अत: कथन 1 सही है।

  • विदेशी संस्थागत निवेश को विदेश प्रत्यक्ष निवेश में शामिल किया जाता है। उल्लेखनीय है कि विदेशी संस्थागत निवेश 24 प्रतिशत की समग्र सीमा की शर्त के अधीन हैं। अत: कथन (2) सही है।
  • इसी तरह वैश्विक डिपॉजिटरी रसीदें (GDR) भारत में विदेशी प्रत्यक्ष निवेश का साधन हैं। अत: कथन 3 सही है।
  • अनिवासी विदेशी जमा भुगतान संतुलन खाते में एक ऋण सृजन प्रवाह है। अत: कथन (4) सही नहीं है। अत: विकल्प (a) सही होगा।

38. निम्नलिखित कथनों पर विचार कीजिये-

किसी मुद्रा के अवमूल्यन का प्रभाव यह है कि वह अनिवार्य रूप से

  1. विदेशी बाज़ारों में घरेलू निर्यातों की प्रतिस्पर्धात्मकता को बढ़ाता है।
  2. घरेलू मुद्रा के विदेशी मूल्य को बढ़ाता है।
  3. व्यापार संतुलन में सुधार लाता है।

उपर्युक्त कथनों में से कौन-सा/से सही है/हैं?

(a) केवल 1  
(b) 1 और 2
(c) केवल 3
(d) 2 और 3

उत्तर:(a) 

व्याख्या: स्थिर विनियम दर प्रणाली (इसके साथ ही प्रबंधित विनिमय दर प्रणाली) के अंतर्गत, जब किसी सरकारी कार्रवाई द्वारा विनिमय दर बढ़ती है अर्थात् घरेलू मुद्रा, विदेशी मुद्रा की तुलना में सस्ती हो जाती है तो इसे मुद्रा अवमूल्यन कहा जाता है। अत: कथन (2) सही नहीं है।

  • मुद्रा के अवमूल्यन से घरेलू मुद्रा का मूल्य विदेशी मुद्रा के मूल्य की तुलना में कम हो जाता है। इसके फलस्वरूप घरेलू निर्यात सस्ता और आयात महँगा हो जाता है। सस्ता घरेलू निर्यात विदेशी बाज़ार में प्रतिस्पर्धात्मकता में वृद्धि करता है। अत: कथन (1) सही है।
  • व्यापार संतुलन एक निश्चित अवधि में किसी देश की वस्तुओं के निर्यात के मूल्य और आयात के मूल्य के बीच का अंतर है। मुद्रा के अवमूल्यन के साथ निर्यात की प्रतिस्पर्धात्मकता में सुधार होता है किंतु व्यापार संतुलन निर्यात और आयात दोनों पर निर्भर करता है और यह अनिवार्य नहीं है कि मुद्रा के अवमूल्यन से व्यापार संतुलन में सुधार हो। अत: कथन 3 सही नहीं है। अत: विकल्प (a) सही होगा।

39. भारत में काले धन के सृजन के निम्नलिखित प्रभावों में से कौन-सा भारत सरकार की चिंता का प्रमुख कारण है?

(a) स्थावर संपदा के व्रय और विलासितायुक्त आवास में निवेश के लिये संसाधनों का अपयोजन
(b) अनुत्पादक गतिविधियों में निवेश और जवाहरात, गहने, सोना इत्यादि का व्रय
(c) राजनीतिक दलों को बड़े चंदे एवं क्षेत्रवाद का विकास
(d) कर अपवंचन के कारण राजकोष में राजस्व की हानि

उत्तर:(d)

व्याख्या: राष्ट्रीय लोक वित्त एवं नीति संस्थान(एनआईपीएफपी) के अनुसार, ‘काला धन वह धन है जिस पर कर की देनदारी तो बनती है लेकिन उसकी जानकारी कर विभाग को नहीं दी जाती है। काला धन सरकार की आय में रुकावटें तो उत्पन्न करता ही है, साथ ही देश के सीमित वित्तीय साधनों को अवांछित दिशाओं में मोड़ देता है। इसमें अवैध तरीकों से अर्जित किया गया धन तथा कर योग्य, वह धन जिस पर कर न दिया गया हो, को काले धन की श्रेणी में रखा जाता है।

  • काला धन के सृजन का एक स्रोत आपराधिक गतिविधियाँ भी हैं।  इसमें अपहरण, तस्करी, नशीली दवाएँ, अवैध खनन, जालसाज़ी और घोटाले आदि आपराधिक गतिविधियाँ आती हैं। इसके अलावा भ्रष्टाचार जैसे रिश्वतखोरी और चोरी भी काले धन का प्रमुख स्रोत हैं। काले धन की उत्पत्ति का दूसरा स्रोत, कर अपवंचन है। इसके तहत यदि किसी व्यक्ति की वार्षिक आय आयकर के अंतर्गत है तथा वह आयकर की राशि को बचाने के लिये अपनी वास्तविक आय के स्थान पर कम आय को दर्शाता है तो वास्तविक आय और घोषित आय के बीच का अंतर काला धन कहलाता है। उल्लेखनीय है कि देश में काले धन के पैदा होने का सबसे बड़ा कारण यही है। विकल्प (a), (b) और (c) काले धन के सृजन और निवेश के तरीके हैं जबकि विकल्प (d) काले धन के निर्माण का प्रभाव है। अत: विल्कप (d) सही होगा।

40. निम्नलिखित में से कौन-सा अपने प्रभाव मेें सर्वाधिक मुद्रास्फीतिकारक हो सकता है?

(a) सार्वजनिक ऋण की चुकौती
(b) बजट घाटे के वित्तीयन के लिये जनता से उधार लेना
(c) बजट घाटे के वित्तीयन के लिये बैंकों से उधार लेना
(d) बजट घाटे के वित्तीयन के लिये नई मुद्रा का सृजन करना

उत्तर:(d)

व्याख्या: किसी भी कदम को स्फीतिकारी (inflationary)  तब माना जाता है जब उसके परिणामस्वरूप बाज़ार में मुद्रा की आपूर्ति में वृद्धि होती है। विकल्प (b) और (c) सही नहीं हैं क्योंकि बजट घाटे के वित्तीयन के लिये जनता से उधार अथवा बैंकों से उधार लेने का अर्थ है बाज़ार से तरलता खींचना, न कि बढ़ाना। 

  • विकल्प (a) के अनुसार सार्वजनिक ऋण की चुकौती से बाज़ार में मुद्रा आपूर्ति में वृद्धि ज़रूर होती है और इसमें स्फीतिकारी प्रभाव की संभावना भी रहती है किंतु सार्वजनिक ऋण की चुकौती से सरकार की सार्वजनिक व्यय करने की क्षमता कम होने की संभावना रहती है जो स्फीतिकारी प्रभाव को संतुलित भी कर सकता है। अत: विकल्प (a) सही नहीं होगा।
  • लेकिन सर्वाधिक स्फीतिकारी होने की दृष्टि से विकल्प (d) सही है, क्योंकि नई मुद्रा सर्वाधिक तरल होती है एवं यह वस्तुओं अथवा सेवाओं के उत्पादन में कोई वृद्धि किये बिना ही अर्थव्यवस्था में मुद्रा की आपूर्ति में वृद्धि कर देती है।

41. निम्नलिखित में से किसका उपयोग प्राकृतिक मच्छर प्रतिकर्षी तैयार करने में किया जाता है?

(a) कांग्रेस घास  
(b) एलिफैंट घास
(c) लेमन घास  
(d)  नट घास

उत्तर:(c) 

व्याख्या: कांग्रेस घास मूल रूप से अमेरिका, मेक्सिको तथा वेस्टइंडीज क्षेत्र में पाई जाती हैैै। भारत, ऑस्ट्रलिया तथा अप्रीका में यह आक्रमणकारी प्रजाति के रूप में पाई जाती है। यह मच्छरों की शरणस्थली है। यह पशुओं में त्वचा संबधी समस्याओं एंव मनुष्यों में श्वसन संबंधी समस्याओं हेतु उत्तरदायी है।

  • एलिफेंट घास को ‘नेपियर घास’ तथा ‘युगाण्डा घास’ के नाम से भी जाना जाता है। यह मूलत: अप्रीका में पाई जाती है। इसका उपयोग फसलों में लगने वाले कीड़ों को नष्ट करने में किया जाता है।
  • लेमन घास में एस्कॉर्बिक एसिड पाया जाता है जो रोग प्रतिरोधक क्षमता बढ़ाने में सहायक होता है। इसमें सिट्रोनेला नामक तेल पाया जाता है जिसकी तेज़ सुंगध के कारण इसका प्रयोग मच्छर प्रतिकर्षी तैयार करने में किया जाता है। अत: विकल्प (c) सही है।
  • नट घास को ‘मोथा’ के नाम से भी जाना जाता है। यह खेतों के लिये सर्वाधिक खराब खतपरवार के लिये जानी जाती है, जो फसलों की बर्बादी के लिये प्रमुख रूप से उत्तरदायी है। हालँाकि इसके औषधीय गुणों के कारण इसका प्रयोग रोग प्रतिरोधक क्षमता बढ़ाने के लिये भी किया जाता है।

42. जीवों के निम्नलिखित प्रकारों पर विचार कीजिये-

1. कॉपिपोड        
2. साइनोबैक्टीरिया
3. डायटम         
4. फोरैमिनिफेरा

उपर्युक्त में से कौन-से जीव महासागरों की आहार  शृंखलाओं में प्राथमिक उत्पादक हैं?

(a) 1 और 2                     
(b) 2 और 3
(c) 3 और 4      
(d)  1 और 4

उत्तर:(b) 

व्याख्या: हरे पेड़-पौधे,कुछ खास जीवाणु एवं शैवाल जो सूर्य प्रकाश की उपस्थिति में अपना भोजन स्वयं बना सकते हैं,स्वपोषी अथवा प्राथमिक उत्पादक कहलाते हैं। महासागरों की आहार  शृंखलाओं में पादप्लवक प्रमुख उत्पादक होते हैं। इसमें साइनोबैक्टीरिया (नील हरित शैवाल) एवं डायटम क्लोरोफिल की उपस्थिति में अपना भोजन स्वयं बनाते है, वहीं कॉपीपोड तथा फोरेमिनिफेरा उपभोक्ता की श्रेणी में आते हैं। अत: विकल्प (b) सही है।

43. निम्नलिखित प्राणियों पर विचार कीजिये-

  1. जाहक (हेज्हॉग)
  2. शैलमूषक (मारमॉट)
  3. वज्रशल्क (पैंगोलिन)

उपर्युक्त में से कौन-सा/से जीव परभक्षियों द्वारा पकड़े जाने की संभावना को कम करने के लिये, स्वयं को लपेटकर अपने सुभेद्य अंगों की रक्षा करता है/करते हैं?

(a) 1 और 2
(b) केवल 2
(c) केवल 3 
(d) 1 और 3

उत्तर:(d)

व्याख्या: जाहक (हेज्हॉग) और वज्रशल्क (पैंगोलिन) ऐसे जीव हैं जो परभक्षियों द्वारा पकड़े जाने की संभावना को कम करने के लिये स्वयं को लपेटकर अपनी रक्षा करते हैं। हेज्हॉग एक छोटा स्तनपायी है और पैंगोलिन पृथ्वी पर पाया जाने वाला एकमात्र सशल्क स्तनपायी जीव है। विश्व में पाई जाने वाली पैंगोलिन की आठ प्रजातियों में से दो,चीनी पैंगोलिन तथा इंडियन पैंगोलिन भारत में पाई जाती हैं। ये अधिकतर पूर्वोत्तर भारत में देखी जा सकती हैं। वहीं शैलमूषक (मारमॉट) गिलहरी परिवार के सबसे भारी सदस्य हैं जो गर्मियों के दौरान सक्रिय होते हैं। अत: विकल्प (d) सही है।

44.वनों पर न्यूयॉर्क घोषणा (न्यूयॉर्क डिक्लेरेशन ऑन फॉरेस्ट्स)’ के संदर्भ में, निम्नलिखित में से कौन-से कथन सही हैं?

  1. 2014 में, संयुक्त राष्ट्र जलवायु शिखर सम्मेलन में पहली बार इसका समर्थन किया गया था।
  2. इसमें वन के ह्रास को रोकने के लिये एक वैश्विक समय-रेखा का समर्थन किया गया।
  3. यह वैध रूप से बाध्यकारी अंतर्राष्ट्रीय घोषणा है।
  4. यह सरकारों, बड़ी कंपनियों और देशीय समुदायों द्वारा समर्थित है।
  5. भारत, इसके प्रारंभ के समय, हस्ताक्षरकर्त्ताओं में से एक था।

नीचे दिये गए कूट का प्रयोग कर सही उत्तर चुनिये-

(a) 1, 2 और 4  
(b) 1, 3 और 5
(c) 3 और 4               
(d) 2 और 5

उत्तर:(a) 

व्याख्या: वनों पर 2014 की न्यूयॉर्क घोषणा पर 200 से अधिक हस्ताक्षर किये गए थे, जिनमें विभिन्न देशों और कंपनियों के अलावा पर्यावरण समूह भी शामिल थे। न्यूयॉर्क घोषणा में वर्ष 2020 तक वैश्विक प्राकृतिक वन हानि को आधा करना तथा वर्ष 2030 तक इस हानि को समाप्त करना शामिल है। इसके अंतर्गत वनों के ह्रास को रोकने के लिये एक वैश्विक समय-रेखा का समर्थन किया गया है। अत: विकल्प (a) सही है।

45.तंत्रिका अपह्रास (न्यूरोडीजेनेरेटिव) समस्याओं के लिये उत्तरदायी माने जाने वाले मैग्नेटाइट कण पर्यावरणीय प्रदूषकों के रूप में निम्नलिखित में से किनसे उत्पन्न होते हैं?

  1. मोटरगाड़ी के ब्रेक
  2. मोटरगाड़ी के इंजन
  3. घरों में प्रयोग होने वाले माइक्रोवेव स्टोव
  4. बिजली संयंत्र
  5. टेलीफोन लाइन

नीचे दिये गए कूट का प्रयोग कर सही उत्तर चुनिये-

(a) केवल 1, 2, 3 और 5
(b)  केवल 1, 2 और 4
(c) केवल 3, 4 और 5  
(d)  1, 2, 3, 4 और 5

उत्तर:(d)

व्याख्या: ऑक्सफोर्ड यूनिवर्सिटी के शोधकर्त्ताओं द्वारा मानव मस्तिष्क के ऊतकों में वायु प्रदूषण से संबंधित मैग्नेटाइट या आयरन ऑक्साइड के नैनोकण देखे गए हैं और शोधकर्त्ताओं का मानना हैं कि वे मनोभ्रंश, अल्जाइमर और मिर्गी जैसे तंत्रिका अपह्रास (न्यूरोडीजेनेरेटिव) विकारों के कारणों में से एक हैं। शोधकर्त्ताओं ने मस्तिष्क के ऊतकों में निकल, कोबाल्ट और प्लैटिनम जैसी धातु के नैनोकणों को भी देखा जो बाहरी स्रोत की ओर संकेत करते हैं। ध्यातव्य है कि मैग्नेटाइट एक प्रबल चुंबकीय लौह खनिज है। गोलाकार नैनो-मैग्नेटाइट की रेडॉक्स गतिविधि और सरफेस चार्ज इसे न्यूरोडीजेनेरेटिव बीमारी के कारणों से जोड़ता है। ऐसे मैग्नेटाइट कण प्रमुख रूप से उच्च तापमान पर जीवाश्म ईंधन वाले दहन स्रोत, बिजली स्टेशन, टेलीफोन लाइन, निर्माण प्रक्रिया, वाहन इंजन (विशेष रूप से डीजल) और घरों में प्रयोग होने वाला माइक्रोवेव स्टोव आदि से प्रदूषकों के रूप में उत्पन्न होते हैं। ऐसा प्रदूषण वाहनों के ब्रेक के घर्षण से भी हो सकता है। अत: विकल्प (d) सही है।

46. निम्नलिखित में से कौन-सा जीव निस्यंदक भोजी (फिल्टर फीडर)है?

(a) अशल्क मीन (कैटफिश)
(b)
अष्टभुज (ऑक्टोपस)
(c) सीप (ऑयस्टर)         
(d)  हवासिल (पेलिकन)

उत्तर:(c) 

व्याख्या: फिल्टर फीडर ऐसे जीव हैं (जैसे बड़ी सीपी या बेलन व्हेल) जो अपने शरीर के कुछ हिस्से से गुज़रने वाले पानी की धारा से कार्बनिक पदार्थ या सूक्ष्म जीवों को छानकर अपना भोजन प्राप्त करते हैं। सीप (ऑयस्टर) प्राकृतिक फिल्टर फीडर हैं। इसका अर्थ है कि वे अपने गलफड़ों के माध्यम से पानी पंप करके, भोजन के कणों के साथ-साथ पोषक तत्त्वों, निलंबित तलछट और रासायनिक संदूषकों को प्रग्रहीत कर भोजन करते हैं। ये जल को साफ करने में महत्त्वपूर्ण भूमिका निभाते हैं। अत: विकल्प (c) सही है।

47. निम्नलिखित जैव भू-रासायनिक चक्रों में से किसमें, चट्टानों का अपक्षय चक्र में प्रवेश करने वाले पोषक तत्त्व के निर्मुक्त होने का मुख्य स्रोत है?

(a) कार्बन चक्र                  
(b)  नाइट्रोजन चक्र
(c) फॉस्फोरस चक्र           
(d)  सल्फर चक्र

उत्तर:(c)

व्याख्या: फॉस्फोरस का प्राकृतिक भंडार चटेानों में है जो कि फॉस्फेट के रूप में फॉस्फोरस को संचित किये हुए है। फॉस्फोरस चक्र के अंतर्गत जब चटेानों का क्षय होता है तो थोड़ी मात्रा में ये फॉस्फेट भूमि पर जल में घुल जाते हैं और उन्हें पादपों की जड़ द्वारा अवशोषित कर लिया जाता हैं। शाकाहारी और अन्य जानवर इन तत्त्वों को पादपों से ग्रहण करते हैं। कचरा उत्पादों एवं मृत जीवों को फॉस्फोरस विलेयक जीवाणुओं द्वारा अपघटित करने पर फॅास्फोरस मुक्त होता है। कार्बन चक्र की भाँति पर्यावरण में फॉस्फोरस को श्वसन द्वारा अवमुक्त नहीं किया जाता है। अत: विकल्प (c) सही है।

48. निम्नलिखित में से कौन-से जीव अपरदाहारी (डेट्राइटिवोर) हैं?

1. केंचुआ         
2.  जेलीफिश
3. सहस्रपादी (मिलीपीड) 
4. समुद्री घोड़ा (सीहॉर्स)
5. काष्ठ यूका (वुडलाइस)

नीचे दिये गए कूट का प्रयोग कर सही उत्तर चुनिये-

(a) केवल 1, 2 और 4       
(b)  केवल 2, 3, 4, और 5
(c) केवल 1, 3 और 5        
(d) 1, 2, 3, 4 और 5

उत्तर:(c) 

व्याख्या: अपरदहारी (Detritivore) ऐसे जीव हैं जो मृत या सड़ने वाले पौधों या जानवरों को भोजन के रूप में खाते हैं। इनमें सूक्ष्मजीव जैसे बैक्टीरिया और बड़े जीव जैसे कवक, केंचुआ, कीड़े, सहस्रपादी (Millipedes) और कुछ क्रस्टेशियन (crustacean) जैसे काष्ठ यूका (Woodlice) शामिल हैं। अत: विकल्प (c) सही है।

49. यू.एन.ई.पी. द्वारा समर्थित ‘कॉमन कार्बन मेट्रिक’ को किसलिये विकसित किया गया है?

(a) संपूर्ण विश्व में निर्माण कार्यों के कार्बन पदचिह्न का आकलन करने के लिये
(b) कार्बन उत्सर्जन व्यापार में विश्व भर की वाणिज्यिक कृषि संस्थाओं के प्रवेश हेतु अधिकार देने के लिये
(c) सरकारों को अपने देशों द्वारा किये गए समग्र कार्बन पदचिह्न के आकलन हेतु अधिकार देने के लिये
(d) किसी इकाई समय (यूनिट टाइम) में विश्व में जीवाश्मी ईंधनों के उपयोग से उत्पन्न होने वाले समग्र कार्बन पदचिह्न के आकलन के लिये

उत्तर:(a) 

व्याख्या: संयुक्त राष्ट्र पर्यावरण कार्यक्रम (यूएनईपी) द्वारा समर्थित ‘कॉमन कार्बन मेट्रिक को संपूर्ण विश्व में निर्माण कार्यों के कार्बन पदचिह्न  को आकलित करने लिये विकसित किया गया है। यह दुनिया भर की इमारतों से उत्सर्जन का लगातार मूल्यांकन और तुलना करने एवं सुधारों को मापने की अनुमति प्रदान करता है। इमारतों से लगातार, मापने योग्य, रिपोर्ट करने योग्य और सत्यापन योग्य ग्रीनहाउस गैस उत्सर्जन में कमी के लिये इनकी आवश्यकता होती है। इन मेट्रिक्स को अलग-अलग इमारतों या इमारतों के समूहों में ऊर्जा के उपयोग को मापने के लिये लागू किया जा सकता है। अत: विकल्प (a) सही है।

50. निम्नलिखित समूहों में से किन में ऐसी जातियाँ होती हैं, जो अन्य जीवों के साथ सहजीवी संबंध बना सकती हैं?

  1. नाइडेरिया
  2. कवक (पंजाई)
  3. आदिजंतु (प्रोटोजोआ)

नीचे दिये गए कूट का प्रयोग कर सही उत्तर चुनिये-

(a) केवल 1 और 2           
(b)  केवल 2 और 3
(c) केवल 1 और 3           
(d) 1, 2 और 3

उत्तर:(d)

व्याख्या: सहजीवी संबंध (Symbiotic Relationship) दो प्राणियों में परस्पर लाभजनक, आंतरिक साझेदारी है। यह सहभागिता दो पौधों या दो जंतुओं के बीच, या पौधे और जंतु के पारस्परिक संबंध में हो सकती है। अधिकांश कवक (फंजाई) परपोषित होते हैं। ये शैवाल तथा लाइकेन के साथ एवं उच्चवर्गीय पौधों के साथ कवक मूल बना कर भी रह सकते हैं। ऐसे कवक सहजीवी कहलाते हैं। नाइडेरिया, जिसे सीलेन्टरेटा भी कहा जाता है। इनमें मूंगा, हाइड्रा, जेलीफिश, समुद्री एनीमोन, आदि शामिल हैं। नाइडेरिया और शैवाल के बीच सहजीवी संबंध पाया जाता है। साथ ही, दीमक का प्रोटोजोआ के साथ सहजीवी संबंध होता है जो इस कीट की आँत में रहते हैं। दीमक अपना भोजन सेल्यूलोज के रूप में प्राप्त करते हैं, जिसे पचा पाने की क्षमता इनमें नहीं होती। प्रोटोजोआ के भीतर सेल्यूलोज को पचाने की क्षमता से दीमक को लाभ होता है। अत: कथन (d) सही है।

51. भारतीय संविधान के अंतर्गत धन का केंद्रीकरण किसका उल्लंघन करता है?

(a) समता का अधिकार   
(b)
  राज्य की नीति के निदेशक तत्त्व
(c) स्वातंत्र्य का अधिकार 
(d)  कल्याण की अवधारणा

उत्तर:(b) 

व्याख्या:संविधान के भाग-3 में अनुच्छेद 36-51 के मध्य ‘राज्य की नीति के निदेशक तत्त्व वर्णित हैं। भाग-3 के ही अंतर्गत अनुच्छेद 39 के खंड (ग) में प्रावधान है कि राज्य अपनी नीति का, विशिष्टतया, इस प्रकार संचालन करेगा कि सुनिश्चित रूप से आर्थिक व्यवस्था इस प्रकार चले जिससे धन और उत्पादन-साधनों का सर्वसाधारण के लिये अहितकारी संकेंद्रण न हो।  अतः विकल्प (b) सही उत्तर होगा।

52. भारत में संपत्ति के अधिकार की क्या स्थिति है?

(a) यह विधिक अधिकार है, जो केवल नागरिकों को प्राप्त है
(b) यह विधिक अधिकार है, जो किसी भी व्यक्ति को प्राप्त है
(c) यह मूल अधिकार है, जो केवल नागरिकों को प्राप्त है
(d) यह न तो मूल अधिकार है, न ही विधिक अधिकार

उत्तर:(b)

व्याख्या: भारतीय संविधान के अनुच्छेद 300क में प्रावधान है कि किसी व्यक्ति को उसकी संपत्ति से विधि के प्राधिकार से ही वंचित किया जाएगा, अन्यथा नहीं। इस अनुच्छेद से स्पष्ट हो जाता है कि भारत में संपत्ति का अधिकार एक विधिक अधिकार है, जो किसी भी व्यक्ति को प्राप्त है। अत: विकल्प (b) सही उत्तर होगा।

53. 26 जनवरी, 1950 को भारत की वास्तविक सांविधानिक स्थिति क्या थी?

(a) लोकतंत्रात्मक गणराज्य
(b) संपूर्ण प्रभुत्व-संपन्न लोकतंत्रात्मक गणराज्य
(c) संपूर्ण प्रभुत्व-संपन्न पंथनिरपेक्ष लोकतंत्रात्मक गणराज्य
(d) संपूर्ण प्रभुत्व-संपन्न समाजवादी पंथनिरपेक्ष लोकतंत्रात्मक गणराज्य

उत्तर:(b)

व्याख्या:  26 जनवरी, 1950 को लागू हुए भारतीय संविधान की उद्देशिका में भारत को ‘संपूर्ण प्रभुत्व-संपन्न लोकतंत्रात्मक गणराज्य के रूप में घोषित किया था। अर्थात्, तब के अनुसार भारत की वास्तविक सांविधानिक स्थिति ‘संपूर्ण प्रभुत्व-संपन्न लोकतंत्रात्मक गणराज्य की ही थी। अत: विकल्प (b) सही उत्तर होगा।

नोट: हालाँकि 1976 में किये गए संविधान के 42वें संशोधन के पश्चात् प्रस्तावना में ‘समाजवादी, ‘पंथनिरपेक्ष और ‘अखंडता शब्द जोड़ दिये गए, जिससे भारत की सांविधानिक स्थिति में परिवर्तन हुआ।

54. सांविधानिक सरकार का आशय क्या है?

(a) किसी राष्ट्र की परिसंघीय संरचना वाली एक प्रतिनिधि सरकार
(b) कोई सरकार, जिसके प्रमुख के पास नाममात्र की शक्तियाँ हाें
(c) कोई सरकार, जिसके प्रमुख के पास वास्तविक शक्तियाँ हों
(d) कोई सरकार, जो संविधान की सीमाओं से परिबद्ध हो

उत्तर:(d) 

व्याख्या: परिभाषा से, सांविधानिक सरकार वह सरकार है, जो संविधान की सीमाओं से परिबद्ध हो। अत: विकल्प (d) सही उत्तर होगा। उल्लेखनीय है कि सरकार के प्रमुख  के पास नाममात्र की शक्तियाँ होना ‘संसदीय प्रणाली की सरकार का अभिलक्षण है, जबकि सरकार के प्रमुख के पास वास्तविक शक्तियाँ होना ‘अध्यक्षीय प्रणाली की सरकार का अभिलक्षण है। किसी राष्ट्र की परिसंघीय संरचना वाली प्रतिनिधि सरकार की उपस्थिति ‘परिसंघ (Federation) की विशेषता है।

55. भारत के संदर्भ में ‘हल्बी, हो और कुई’ पद किससे संबंधित हैं?

(a) पश्चिमोत्तर भारत का नृत्यरूप       
(b) वाद्ययंत्र
(c) प्रागैतिहासिक गुफा चित्रकला         
(d)  जनजातीय भाषा

उत्तर:(d) 

व्याख्या:‘हल्बी उड़ीसा में बोली जाने वाली जनजातीय भाषा है। ‘हो, मुंडा परिवार की जनजातीय भाषा है। जबकि ‘कुई द्रविड़ परिवार की जनजातीय भाषा है जिसको बोलने वाले उड़ीसा में रहते हैं। अत: विकल्प (d) सही है।

56. भारतरत्न और पँ पुरस्कारों के संबंध में, निम्नलिखित कथनों पर विचार कीजिये:

  1. भारतरत्न और पँ पुरस्कार भारत के संविधान के अनुच्छेद-18(1) के अंतर्गत उपाधियाँ हैं।
  2. वर्ष 1954 में प्रारंभ किये गए पँ पुरस्कारों को केवल एक बार निलंबित किया गया था।
  3. किसी वर्ष-विशेष में भारतरत्न पुरस्कारों की अधिकतम संख्या पाँच तक सीमित है।

उपर्युक्त कथनों में से कौन-से सही नहीं हैं?

(a) केवल 1 और 2           
(b)  केवल 2 और 3
(c) केवल 1 और 3           
(d) 1, 2 और 3

व्याख्या: संविधान का अनुच्छेद 18 (1) यह प्रावधान करता है कि राज्य, सेना या विद्या संबंधी सम्मान के सिवाय और कोई उपाधि प्रदान नहीं करेगा। 1995 में सर्वोच्च न्यायालय द्वारा बालाजी राघवन मामले में दिये गए निर्णय में यह स्पष्ट किया गया कि भारतरत्न व पँ पुरस्कार संविधान के अनुच्छेद 18 (1) में वर्णित उपाधियाँ नहीं हैं। अत: कथन 1 सही नहीं है।

  • उल्लेखनीय है कि वर्ष 1954 में आरंभ किये जाने के पश्चात् ऐसा दो बार (1978-791993-97) हुआ कि पँ पुरस्कार वितरित नहीं किये गए, क्योंकि इन्हें निलंबित किया गया था। अत: कथन 2 सही नहीं है।
  • किसी वर्ष-विशेष में अधिकतम 3 भारतरत्न पुरस्कार दिये जा सकते हैं। अत: कथन 3 सही नहीं है। इस प्रकार विकल्प (d) सही उत्तर होगा।

57. निम्नलिखित कथनों पर विचार कीजिये:

कथन 1: संयुक्त राष्ट्र पूंजी विकास निधि (यू.एन.सी.डी.एफ.) और आर्बर डे फाउंडेशन ने हाल ही में हैदराबाद को विश्व के 2020 वृक्ष नगर की मान्यता प्रदान की है।

कथन 2: शहरी वनों को बढ़ाने और संपोषित करने के प्रति प्रतिबद्धता को देखते हुए हैदराबाद का एक वर्ष के लिये इस मान्यता हेतु चयन किया गया है।

उपर्युक्त कथनों के संदर्भ में, निम्नलिखित में से कौन-सा सही है?

(a) कथन 1 और कथन 2 दोनों सही हैं और कथन 2, कथन 1 की सही व्याख्या है
(b) कथन 1 और कथन 2 दोनों सही हैं, किंतु कथन 2, कथन 1 की सही व्याख्या नहीं है
(c) कथन 1 सही है, किंतु कथन 2 सही नहीं है
(d) कथन 1 सही नहीं है, किंतु कथन 2 सही है

उत्तर:(d) 

व्याख्या: हैदराबाद भारत का वह एकमात्र शहर है, जिसे आर्बर डे फाउंडेशन और संयुक्त राष्ट्र के खाद्य और कृषि संगठन (FAO) द्वारा एक वर्ष के लिये ‘2020 विश्व के वृक्ष नगर के रूप में मान्यता दी गई है। हैदराबाद को 63 देशों के 119 अन्य शहरों के साथ यह मान्यता प्रदान की गई है। स्वस्थ और खुशहाल नगरों के निर्माण में नगरीय वनों को बढ़ाने और संपोषित करने के प्रति उनकी प्रतिबद्धता के लिये इन्हें वृक्ष नगर की मान्यता दी गई है। ध्यातव्य है कि संयुक्त राज्य अमेरिका, कनाडा और यूनाइटेड किंगडम इस सूची में अधिकतम नगरों वाले देश हैं, जिनमें क्रमश: 38] 15 और 11 नगर हैं। अत: कथन (d) सही है।

58. वर्ष 2000 में प्रारंभ किये गए लॉरियस विश्व खेल पुरस्कार (लॉरियस वर्ल्ड स्पोर्ट्स अवार्ड) के संबंध में निम्नलिखित कथनों पर विचार कीजिये:

  1. अमेरिकी गोल्फ खिलाड़ी टाइगर वुड्स इस पुरस्कार का सर्वप्रथम विजेता थे।
  2. अब तक यह पुरस्कार अधिकतर ‘फॉर्मूला वन के खिलाड़ियों को मिला है।
  3. अन्य खिलाड़ियों की तुलना में रॉजर पेडरर को यह पुरस्कार सर्वाधिक बार मिला है।

उपर्युक्त कथनों में से कौन-से सही हैं?

(a) केवल 1 और 2           
(b)  केवल 2 और 3
(c) केवल 1 और 3           
(d) 1, 2 और 3

उत्तर:(c)

व्याख्या: लॉरियस विश्व खेल पुरस्कार का आयोजन पहली बार 2000 में किया गया था। इसके अंतर्गत छह श्रेणियों (पुरुष खिलाड़ी, महिला खिलाड़ी, टीम, ब्रेकथ्रू, कमबैक और एक्शन) में पुरस्कार दिये जाते हैं।

  • वर्ष 2000 में विश्व के सर्वश्रेष्ठ पुरुष खिलाड़ी के लिये लॉरियस खेल पुरस्कार गोल्फर टाइगर वुड्स ने जीता था। अत: कथन 1 सही है।
  • अब तक वितरित किये गए 21 पुरस्कारों में से अधिकतर ‘लॉन टेनिस के खिलाड़ियों ने जीते हैं। अत: कथन 2 सही नहीं है।
  • स्विट्ज़रलैंड के टेनिस खिलाड़ी रॉजर फेडरर ने अब तक सर्वाधिक 5 बार यह पुरस्कार जीता है। अत: कथन 3 सही है। विकल्प (c) सही उत्तर होगा।

59. 32वें ग्रीष्मकालीन ओलंपिक के संदर्भ में, निम्नलिखित कथनों पर विचार कीजिये:

  1. इस ओलंपिक का आधिकारिक आदर्श वाक्य ‘एक नई दुनिया (ए न्यू वर्ल्ड)’ है।
  2. इस ओलंपिक में स्पोर्ट क्लाइंबिंग, सर्पिंग, स्केटबोर्डिंग, कराटे तथा बेसबॉल को शामिल किया गया है।

उपर्युक्त कथनों में से कौन-सा/से सही है/हैं?

(a) केवल 1                        
(b)  केवल 2
(c) 1 और 2 दोनों              
(d)  न तो 1 और न ही 2

उत्तर:(b) 

व्याख्या: 2021 में टोक्यो में आयोजित किये गए 32वें ग्रीष्मकालीन ओलंपिक खेलों का आधिकारिक आदर्श वाक्य ‘यूनाइटेड बाई इमोशन (United by Emotion) था। यह आदर्श वाक्य विविध पृष्ठभूमि के लोगों को एक साथ लाने की खेल की शक्ति पर ज़ोर देता है और उनके लिये इस तरह से जुड़ना और जश्न मनाना संभव बनाता है, जो उनके मतभेदों से परे हो। अत: कथन 1 सही नहीं है।

  • इस ओलंपिक में कुल छह नए खेल-स्पोर्ट क्लाइंबिंग, सर्फ़िग, सॉफ्टबॉल, बेसबॉल, कराटे तथा स्केटबोर्ड़िग शामिल किये गए थे। अत: कथन 2 सही है। विकल्प (b) सही उत्तर होगा।

60. आई. सी. सी. वर्ल्ड टेस्ट चैंपियनशिप के संदर्भ में, निम्नलिखित  कथनों पर विचार कीजिये:

  1. अंतिम दौर में पहुँचने वाली टीमों का निर्धारण, उनके द्वारा जीते गए मैचों की संख्या के आधार पर किया गया।
  2. न्यूज़ीलैंड का स्थान इंग्लैंड से ऊपर था, क्योंकि उसने इंग्लैंड की तुलना में अधिक मैच जीते।

उपर्युक्त कथनों में से कौन-सा/से सही है/हैं?

(a) केवल 1                        
(b)  केवल 2
(c)  1 और 2 दोनों              
(d)  न तो 1 और न ही 2

उत्तर:(d) 

व्याख्या: आईसीसी वर्ल्ड टेस्ट चैंपियनशिप के अंतिम दौर में पहुँचने वाली टीमों का निर्धारण उनके द्वारा प्रतिस्पर्धित पॉइंट्स की तुलना में जीते गए पॉइंट्स के प्रतिशत (Percentage of Points Contested) के आधार पर किया गया था। इसी आधार पर 2019&2021 के बीच टेस्ट सीरीज़ खेलने वाली टीमों की रैंकिंग तैयार की गई थी। टूर्नामेंट के अंत (जून 2021) में रैंकिंग में शीर्ष 2 स्थानों पर रही टीमों के बीच फाइनल खेला गया। उदाहरण- भारत ने संपूर्ण वर्ल्ड टेस्ट चैंपियनशिप में कुल 720 पॉइंट्स के लिये प्रतिस्पर्धा की, जिसमें से उसने 520 पॉइंट्स, अर्थात् 72.2 प्रतिशत पॉइंट्स जीते और रैंकिंग में प्रथम स्थान प्राप्त करते हुए फाइनल में जगह बनाई। अत: कथन 1 सही नहीं है।

  • उल्लेखनीय है कि न्यूज़ीलैंड ने वर्ल्ड टेस्ट चैंपियनशिप में 7 मैच जीते थे, जबकि इंग्लैंड ने इसी दौरान 11 मैच जीते थे। परंतु चूँकि न्यूज़ीलैंड द्वारा कुल प्रतिस्पर्धित पॉइंट्स में से 70 प्रतिशत जीते गए थे जबकि इंग्लैंड ने केवल 64.1 प्रतिशत पॉइंट्स जीते थे, इसी आधार पर न्यूज़ीलैंड का स्थान इंग्लैंड से ऊपर था। अत: कथन 2 सही नहीं है। विकल्प (d) सही उत्तर होगा।

61. निम्नलिखित कथनाें पर विचार कीजिये-

  1. ‘शहर का अधिकार एक सम्मत मानव अधिकार है तथा इस संबंध में, संयुक्त राष्ट्र हैबिटेट (यू. एन. हैबिटेट) प्रत्येक देश  द्वारा की गई प्रतिबद्धताओं को मॉनिटर करता है।
  2. ‘शहर का अधिकार शहर के प्रत्येक निवासी को शहर में सार्वजनिक स्थानों को वापस लेने (रीक्लेम) एवं सार्वजनिक सहभागिता का अधिकार देता है।
  3. ‘शहर का अधिकार का आशय यह है कि राज्य, शहर की अनधिकृत बस्तियों को किसी भी लोक सेवा अथवा सुविधा से वंचित नहीं कर सकता।

उपर्युक्त कथनों में से कौन-सा/से सही है/हैं?

(a) केवल 1                         
(b)  केवल 3
(c) 1 और 2                      
(d) 2 और 3

उत्तर: (c) 

व्याख्या:‘शहर का अधिकार एक सम्मत मानव अधिकार है। यह सभी निवासियों, वर्तमान और भविष्य, स्थायी और अस्थायी, निवास करने, उपयोग करने, कब्ज़ा करने, उत्पादन करने, शासन करने, समावेशी, सुरक्षित और टिकाऊ शहरों, गाँवों और मानव बस्तियों का अधिकार है, जिसे एक पूर्ण और सभ्य जीवन के लिये सामान्य आवश्यकता के रूप में परिभाषित किया गया है। संयुक्त राष्ट्र-पर्यावास (यूएन-हैबीटेट) इस संबंध में प्रत्येक देश द्वारा की गई प्रतिबद्धताओं की निगरानी करता है। यहाँ राज्य, शहर में अनधिकृत कॉलोनियों को किसी भी सार्वजनिक सेवा या सुविधा से वंचित कर सकता है। अत: विकल्प (c) सही है।

62. भारत के संदर्भ में, निम्नलिखित कथनाें पर विचार कीजिये:

  1. न्यायिक हिरासत का अर्थ है कि अभियुक्त संबंधित मजिस्ट्रेट की हिरासत में है और ऐसे अभियुक्त को पुलिस स्टेशन के हवालात में रखा जाता है न कि जेल में।
  2. न्यायिक हिरासत के दौरान, मामले के प्रभारी पुलिस अधिकारी, न्यायालय की अनुमति के बिना संदिग्ध व्यक्ति से पूछताछ नहीं कर सकते।

उपर्युक्त कथनों में से कौन-सा/से सही है/हैं?

(a) केवल 1                         
(b)  केवल 2
(c) 1 और 2 दोनों   
(d)  न तो 1 और न ही 2

उत्तर:(b) 

व्याख्या: भारत के संदर्भ में न्यायिक हिरासत का अर्थ है कि अभियुक्त संबंधित मजिस्ट्रेट की हिरासत में है और उसे जेल में रखा जाता है। अत: कथन 1 सही नहीं है। साथ ही, यदि मामले का प्रभारी पुलिस अधिकारी न्यायिक हिरासत के दौरान किसी संदिग्ध व्यक्ति से पूछताछ करना चाहता है तो उसके द्वारा न्यायालय की अनुमति ली जाना अनिवार्य है। अत: कथन 2 सही है। विकल्प (b) सही उत्तर होगा।

63. भारत के संदर्भ में, निम्नलिखित कथनाें पर विचार कीजिये:

  1. जब एक कैदी पर्याप्त आधार प्रस्तुत करता है, तो ऐसे कैदी को पैरोल मना नहीं किया जा सकता, क्याेंकि वह उसके अधिकार का मामला बन जाता है।
  2. कैदी को पैरोल पर छोड़ने के लिये राज्य सरकारों के अपने नियम हैं।

उपर्युक्त कथनों में से कौन-सा/से सही है/हैं?

(a) केवल 1                        
(b)  केवल 2
(c) 1 और 2 दोनों            
(d)  न तो 1 और न ही 2

उत्तर:(b)

व्याख्या: भारत में कारागारों में सज़ा काट रहे बंदियों को समय-समय पर पैरोल पर छोड़े जाने की व्यवस्था प्रचलित है। सामान्यत: एक बंदी द्वारा पर्याप्त आधार प्रस्तुत किये जाने पर उसे पैरोल दी जा सकती है। हालाँकि पैरोल पर छोड़े जाना किसी भी प्रकार से बंदी के अधिकार का मामला नहीं है और पर्याप्त आधारों के बावजूद प्रशासन उसे पैरोल देने से इनकार कर सकता है। अत: कथन 1 सही नहीं है।

  • संविधान की सातवीं अनुसूची के अंतर्गत दी गई राज्य सूची की प्रविष्टि संख्या 4 ‘‘कारागार, सुधारालय, बोर्स्टल संस्थाएँ और उसी प्रकार की अन्य संस्थाएँ और उनमें निरुद्ध व्यक्ति : कारागारों और अन्य संस्थाओं के उपयोग के लिये अन्य राज्यों से ठहराव’’ है। इससे यह स्पष्ट है कि पैरोल के नियमों सहित कारागारों से संबंधित किसी प्रकार की विधि व नियम बनाना राज्य विधानसभाओं का क्षेत्राधिकार है। अत: कथन 2 सही है व विकल्प (b) सही उत्तर होगा।

64. राष्ट्रीय स्तर पर, अनुसूचित जनजाति और अन्य पारंपरिक वन निवासी (वन अधिकारों की मान्यता) अधिनियम, 2006 के प्रभावी कार्यान्वयन को सुनिश्चित करने के लिये कौन-सा मंत्रालय केंद्रक अभिकरण (नोडल एजेंसी) है?

(a) पर्यावरण, वन और जलवायु परिवर्तन मंत्रालय
(b) पंचायती राज मंत्रालय
(c) ग्रामीण विकास मंत्रालय
(d) जनजातीय कार्य मंत्रालय

उत्तर:(d) 

व्याख्या: अनुसूचित जनजाति और अन्य पारंपरिक वन निवासी (वन अधिकारों की मान्यता) अधिनियम, 2006 की धारा-11 में यह प्रावधान है कि जनजातीय कार्य से संबंधित केंद्र सरकार का मंत्रालय या इस संबंध में केंद्र सरकार द्वारा अधिकृत कोई अधिकारी या प्राधिकरण अधिनियम के प्रावधानों के कार्यान्वयन के लिये नोडल एजेंसी होगा। अत: विकल्प (d) सही उत्तर होगा।

65. कानून को लागू करने के मामले में कोई विधान, जो किसी कार्यपालक अथवा प्रशासनिक प्राधिकारी को अनिर्देशित एवं अनियंत्रित विवेकाधिकार देता है, भारत के संविधान के निम्नलिखित अनुच्छेदों में से किसका उल्लंघन करता है?

(a) अनुच्छेद-14                 
(b)  अनुच्छेद-28
(c) अनुच्छेद-32                 
(d)  अनुच्छेद-44

उत्तर:(a) 

व्याख्या: भारतीय संविधान के अनुच्छेद-14 में यह प्रावधान है कि राज्य, भारत के राज्यक्षेत्र में किसी व्यक्ति को विधि के समक्ष समता से या विधियों के सामान संरक्षण से वंचित नहीं करेगा। यहाँ ‘विधि के समक्ष समता का आशय यह है कि भारत में सभी व्यक्ति विधि के दायरे में समान रूप से आएंगे और किसी को भी कोई विशेषाधिकार प्राप्त नहीं होगा। ऐसे में यदि कोई विधान किसी कार्यपालक अथवा प्रशासनिक अधिकारी को अनिर्देशित एवं अनियंत्रित विवेकाधिकार देता है तो ‘विधि के समक्ष समता के सिद्धांत का उल्लंघन होगा। इस प्रकार विकल्प (a) सही उत्तर होगा।

66. भारतीय राज्य-व्यवस्था में, निम्नलिखित में से कौन-सी अनिवार्य विशेषता है, जो यह दर्शाती है कि उसका स्वरूप संघीय है?

(a)  न्यायपालिका की स्वतंत्रता सुरक्षित है।
(b)  संघ की विधायिका में संघटक इकाइयों के निर्वाचित प्रतिनिधि होते हैं।
(c)  केंद्रीय मंत्रिमंडल में क्षेत्रीय पार्टियों के निर्वाचित प्रतिनिधि हो सकते हैं।
(d)  मूल अधिकार न्यायालयों द्वारा प्रवर्तनीय हैं।

उत्तर:(a)

व्याख्या: संघवाद की अनिवार्य विशेषताओं की बात की जाए तो इसके अंतर्गत-संघ व राज्य के बीच शक्तियों का स्पष्ट संवैधानिक विभाजन; लिखित, कठोर एवं सर्वोच्च संविधान, द्विस्तरीय सरकार तथा स्वतंत्र व निष्पक्ष न्यायपालिका इत्यादि तत्त्व माने जाते हैं। इस प्रकार विकल्प (a) सही उत्तर होगा।

  • उल्लेखनीय है कि संघीय विधायिका में संघटक इकाइयों के निर्वाचित प्रतिनिधि होना संघीय राजव्यवस्था का लक्षण तो है परंतु यह अनिवार्य नहीं है। साथ ही, केंद्रीय मंत्रिमंडल में क्षेत्रीय पार्टियों के निर्वाचित प्रतिनिधि होना व मूल अधिकार न्यायालयों द्वारा प्रवर्तनीय होना राजव्यवस्था के संघीय स्वरूप की अनिवार्य विशेषताओं से प्रत्यक्षत: संबद्ध नहीं हैं।

67. निम्नलिखित में से कौन-सा ‘राज्य’ शब्द को सर्वोत्तम रूप से परिभाषित करता है?

(a)  व्यक्तियों का एक समुदाय, जो बिना किसी बाह्य नियंत्रण के एक निश्चित भू-भाग में स्थायी रूप से निवास करता है और जिसकी एक संगठित सरकार है।
(b)  एक निश्चित भू-भाग के राजनीतिक रूप से संगठित लोग, जो स्वयं पर शासन करने, कानून एवं व्यवस्था को बनाए रखने, अपने नैसर्गिक अधिकारों की रक्षा करने तथा अपनी जीविका के साधनों को सुरक्षित रखने का अधिकार रखते हैं।
(c)  बहुत से व्यक्ति, जो एक निश्चित भू-भाग में बहुत लंबे समय से अपनी संस्कृति, परंपरा और शासन-व्यवस्था के साथ रहते आए हैं।
(d)  एक निश्चित भू-भाग में स्थायी रूप से रह रहा समाज, जिसकी एक केंद्रीय प्राधिकारी तथा केंद्रीय प्राधिकारी के प्रति उत्तरदायी कार्यपालिका और एक स्वतंत्र न्यायपालिका है।

उत्तर:(a) 

व्याख्या: वर्तमान समय में राज्य के चार अनिवार्य तत्त्व माने जाते हैं- जनसंख्या, भूभाग, सरकार व संप्रभुता। दूसरे शब्दों में, यदि व्यक्तियों का एक अथवा अनेक समुदाय (जनसंख्या) एक निश्चित भूभाग में स्थायी रूप से निवास करता है, एक संगठित सरकार वहाँ के शासन-प्रशासन का दायित्व संभालती है तथा उस भूभाग में निवास करने वाले समुदाय या समुदायों पर किसी प्रकार का कोई बाह्य नियंत्रण नहीं है, तो इसे ‘राज्य की संज्ञा दी जाएगी। इस प्रकार उपलब्ध विकल्पों में से विकल्प (a) ‘राज्य शब्द को सर्वोत्तम रूप से परिभाषित करता है।

68. भारतीय न्यायपालिका के संदर्भ में, निम्नलिखित कथनों पर विचार कीजिये:

  1. भारत के राष्ट्रपति की पूर्वानुमति से भारत के मुख्य न्यायमूर्ति द्वारा उच्चतम न्यायालय से सेवानिवृत्त किसी न्यायाधीश को उच्चतम न्यायालय के न्यायाधीश के पद पर बैठने और कार्य करने हेतु बुलाया जा सकता है।
  2. भारत में किसी भी उच्च न्यायालय को अपने निर्णय के पुनर्विलोकन की शक्ति प्राप्त है, जैसा कि उच्चतम न्यायालय के पास है।

उपर्युक्त कथनों में से कौन-सा/से सही है/हैं?

(a) केवल 1                        
(b)  केवल 2
(c)  1 और 2 दोनों  
(d)  न तो 1 और न ही 2

उत्तर:(c) 

व्याख्या: संविधान के अनुच्छेद 128 में यह प्रावधान है कि आवश्यकता पड़ने पर सेवानिवृत्त न्यायाधीशों को भी अल्पकाल के लिये सर्वोच्च न्यायालय के न्यायाधीश के रूप में बैठने और कार्य करने का अनुरोध किया जाता है। (सर्वोच्च न्यायालय के अवकाश प्राप्त न्यायाधीश या उच्च न्यायालयों के अवकाश प्राप्त न्यायाधीश, जो सर्वोच्च न्यायालय का न्यायाधीश नियुक्त होने के लिये सम्यक रूप से अर्ह हों।) सर्वोच्च न्यायालय का मुख्य न्यायाधीश राष्ट्रपति की पूर्व सहमति से ऐसे न्यायाधीश से लिखित अनुरोध कर सकता है। अत: कथन 1 सही है।

  • अनुच्छेद 215 में उच्च न्यायालय को अभिलेख न्यायालय का दर्जा प्राप्त है। उसे अपनी अवमानना के लिये दंड देने की शक्ति सहित ऐसे न्यायालय की सभी शक्तियाँ प्राप्त होंगी। उल्लेखनीय है कि अभिलेख न्यायालय की शक्तियों के अंतर्गत अपने ही निर्णय के पुनर्विलोकन की शक्ति भी शामिल है। अत: कथन 2 सही है। विकल्प (c) सही उत्तर होगा।

69. भारत के संदर्भ में, निम्नलिखित कथनों पर विचार कीजिये:

  1. भारत में केवल एक नागरिक और एक ही अधिवास है।
  2. जो व्यक्ति जन्म से नागरिक हो, केवल वही राष्ट्राध्यक्ष बन सकता है।
  3. जिस विदेशी को एक बार नागरिकता दे दी गई है, किसी भी परिस्थिति में उसे इससे वंचित नहीं किया जा सकता।

उपर्युक्त कथनों में से कौन-सा/से सही है/हैं?

(a) केवल 1                        
(b) केवल 2
(c) 1 और 3                      
(d) 2 और 3

उत्तर:(a) 

व्याख्या: भारत में एकल नागरिकता और एकल अधिवास की व्यवस्था स्वीकार की गई है। अत: कथन 1 सही है।

  • भारत में राष्ट्राध्यक्ष (राष्ट्रपति) देश का कोई भी नागरिक बन सकता है, इस संदर्भ में संबंधित नागरिक को जिस विधि से मान्यता मिली है, उससे जुड़ी कोई बाध्यता आरोपित नहीं की जाती है। अत: कथन 2 सही नहीं है।
  • नागरिकता अधिनियम, 1955 की धारा-10 के अनुसार, ‘‘पंजीकरण, देशीयकरण या केवल संविधान के अनुच्छेद 5(ग) के आधार पर नागरिकता अर्जित करने वाले नागरिकों को केंद्र सरकार आदेश द्वारा नागरिकता से वंचित कर सकती है।’’ इस प्रावधान के आधार पर विदेशियों को भारत की नागरिकता से वंचित किया जा सकता है। अत: कथन 3 सही नहीं है। विकल्प (a) सही उत्तर होगा।

70. निम्नलिखित में से कौन-सा कारक किसी उदार लोकतंत्र में स्वतंत्रता की सर्वोत्तम सुरक्षा को नियत करता है?

(a) एक प्रतिबद्ध न्यायपालिका          
(b) शक्तियों का केंद्रीकरण
(c) निर्वाचित सरकार
(d)  शक्तियों का पृथक्करण

उत्तर:(d)

व्याख्या: किसी उदार लोकतंत्र में स्वतंत्रता की सर्वोत्तम सुरक्षा के लिये यह अनिवार्य है कि शक्तियों का केंद्रीकरण न हो। उल्लेखनीय है कि शक्तियों के पृथक्करण को यदि शासन का आधार बनाया जाता है, तो राज्य की विधायी, कार्यपालिका व न्यायिक शक्तियों का उपयोग पृथक-पृथक संस्थाओं द्वारा स्वायत्त रूप से किया जाता है और इसमें न्यायपालिका स्वतंत्र होती है, जिससे शासन व्यवस्था के किसी एक अंग द्वारा स्वतंत्रता पर आघात किये जाने की स्थिति में अन्य अंग नागरिकों की स्वतंत्रता के संरक्षण के लिये आगे आते हैं। इस प्रकार शक्तियों के पृथक्करण के माध्यम से उनके केंद्रीकरण की प्रवृत्ति को हतोत्साहित किया जाता है तथा इससे ही स्वतंत्रता की सर्वोत्तम रक्षा की जा सकती है। अत: विकल्प (d) सही उत्तर होगा।

71. सवाना की वनस्पति में बिखरे हुए छोटे वृक्षों के साथ घास के मैदान होते हैं, किंतु विस्तृत क्षेत्र में कोई वृक्ष नहीं होते हैं। ऐसे क्षेत्रों में वन विकास सामान्यत: एक या एकाधिक या कुछ परिस्थितियों के संयोजन के द्वारा नियंत्रित होता है। ऐसी परिस्थितियाँ निम्नलिखित में से कौन-सी हैं?

  1. बिलकारी प्राणी और दीमक
  2. अग्नि
  3. चरने वाले तृणभक्षी प्राणी (हर्बिवोर्स)
  4. मौसमी वर्षा
  5. मृदा के गुण

नीचे दिये गए कूट का प्रयोग कर सही उत्तर चुनिये-

(a) 1 और 2                      
(b) 4 और 5
(c)  2, 3 और 4      
(d) 1, 3 और 5

उत्तर:(c) 

व्याख्या: सवाना की वनस्पति में बिखरे हुए छोटे वृक्ष और घास के मैदान इसकी प्रमुख विशेषता है। उर्वरता की दृष्टि से ये क्षेत्र कम समृद्ध होते हैं। इन क्षेत्रों में वन विकास का प्रमुख कारक मौसमी वर्षा है।

  • यहाँ वर्षा आमतौर पर ग्रीष्मकाल में होती है तथा शुष्क और आर्द्र मौसम क्रम से आते हैं। इन क्षेत्रों में शुष्कता ज़्यादा होने से आग लगने का खतरा बढ़ जाता है, परिणामस्वरूप वृक्षों का विकास नहीं हो पाता।
  • यहाँ चरने वाले तृणभक्षी प्राणी (हर्बिवोर्स) भी वनस्पति के विकास को प्रभावित करते हैं। विदित हो कि दीमक मुख्य रूप से उष्णकटिबंधीय और उपोष्णकटिबंधीय क्षेत्रों में लकड़ी और पादप पदार्थों का पुन: उपयोग करने वाले अत्यधिक पारिस्थितिक महत्त्व वाले कीट हैं। अत: विकल्प (c) सही है।

72. पृथ्वी ग्रह पर जल के संदर्भ में, निम्नलिखित कथनों पर विचार कीजिये:

  1. नदियों और झीलों में जल की मात्रा, भू-जल की मात्रा से अधिक है।
  2. ध्रुवीय हिमच्छद और हिमनदों में जल की मात्रा, भू-जल की मात्रा से अधिक है।

उपर्युक्त कथनों में से कौन-सा/से सही है/हैं?

(a) केवल 1       
(b)  केवल 2
(c)  1 और 2 दोनों   
(d)  न तो 1 और न ही 2

उत्तर:(b)

व्याख्या: धरातल का लगभग दो-तिहाई से भी अधिक भाग जल से ढँका हुआ है। जलमंडल में महासागर, झील, नदियाँ, भूमिगत जल, हिमनदियाँ आदि सभी सम्मिलित होते हैं। इनमें महासागर सबसे बड़े जलखंड है। जल का लगभग 97 प्रतिशत भाग महासागरों में पाया जाता है,जो लवणीय होने के कारण पीने योग्य नहीं होता है। शेष 3 प्रतिशत का अधिकांश भाग हिमचादरों, हिमनदियों में और फिर भूमिगत जल के रूप में पाया जाता है। नदियों और झीलों में जल की मात्रा अपेक्षाकृत कम है। अत: विकल्प (b) सही है।

73. निम्नलिखित कथनों पर विचार कीजिये:

  1. मोरिंगा (सहजन वृक्ष) एक फलीदार सदापर्णी वृक्ष है।
  2. इमली का पेड़ दक्षिण एशिया का स्थानिक वृक्ष है।
  3. भारत में अधिकांश इमली लघु वनोत्पाद के रूप में संगृहीत की जाती है।
  4. भारत इमली और मोरिंगा के बीज निर्यात करता है।
  5. मोरिंगा और इमली के बीजों का उपयोग जैव ईंधन के उत्पादन में किया जा सकता है।

उपर्युक्त कथनों में से कौन-से सही हैं?

(a)  1, 2, 4 और 5                
(b) 3, 4 और 5
(c)  1, 3 और 4      
(d) 1, 2, 3 और 5

उत्तर:(b) 

व्याख्या: मोरिंगा (सहजन वृक्ष) एक फलीदार पर्णपाती वृक्ष है। यह वृक्ष उष्णकटिबंधीय और उपोष्णकटिबंधीय क्षेत्रों में व्यापक रूप से खेती की जाने वाली फसलों में से एक है।

  • मोरिंगा के बीज का तेल एक उच्च ऑक्सीकरणी स्थिरता (Oxidative Stability) प्रदर्शित करता है और इसकी तापीय स्थिरता अन्य तेल फसलों जैसे सूरजमुखी तेल, सोयाबीन तेल आदि से अधिक होती है। मोरिंगा जैव ईंधन (बायोडीजल) को लंबे समय तक संगृहीत किया जा सकता है और यह परिवहन के लिये सुरक्षित है।
  • इसके अलावा, इमली अफ्रीका में उष्णकटिबंधीय क्षेत्र का स्थानिक वृक्ष है। इस खटेे-मीठे फल का उपयोग बड़े पैमाने पर खाद्य व पेय पदार्थों तथा पारंपरिक दवाओं में किया जाता है। इमली (बीजसहित) को लघु वनोपज के रूप में वर्गीकृत किया गया है।
  • इमली के बीज के तेल का उपयोग भी जैव ईंधन के उत्पादन में किया जा सकता है, जो कार्बन उत्सर्जन को कम करने में सहायक हो सकता है। उल्लेखनीय है कि भारत से लगभग 60 देशों में इमली का निर्यात किया जाता है। साथ ही, दुनिया की लगभग 80 प्रतिशत मोरिंगा मांग की आपूर्ति भारत करता है। अत: विकल्प (b) सही है।

74. भारत में काली कपास मृदा की रचना, निम्नलिखित में से किसके अपक्षयण से हुई है?

(a) भूरी वन मृदा
(b) विदरी (फिशर) ज्वालामुखीय चट्टान
(c) ग्रेनाइट और शिस्ट
(d) शेल और चूना-पत्थर

उत्तर:(b)

व्याख्या: काली मृदा का निर्माण दरारी उद्भेदन से निकले लावा पदार्थों (बेसाल्ट चटेान) के विखंडन से हुआ है। यह कपास की खेती के लिये अधिक उपयोगी एवं विख्यात है, इसलिये इसे ‘काली कपासी मृदा या ‘रेगुर के नाम से भी जाना जाता है। इसके अतिरिक्त इसे ‘उष्ण कटिबंधीय चेरनोजम’ व ‘ट्रॉपिकल ब्लैक अर्थ भी कहते हैं। उत्तर प्रदेश में इस मृदा को ‘करेल की संज्ञा दी जाती है। कपास के अतिरक्त यह मृदा गन्ना, गेहूँ, प्याज और फलों की खेती करने के लिये अनुकूल है।

अत: विकल्प (b) सही है।

75. ‘पुन:संयोजित (रीकॉम्बिनेंट) वेक्टर वैक्सीन’ से संबंधित हाल के विकास के संदर्भ में, निम्नलिखित कथनों पर विचार कीजिये:

  1. इन वैक्सीनों के विकास में आनुवंशिक इंजीनियरी का प्रयोग किया जाता है।
  2. जीवाणुओं और विषाणुओं का प्रयोग रोगवाहक (वेक्टर) के रूप में किया जाता है।

उपर्युक्त कथनों में से कौन-सा/से सही है/हैं?

(a) केवल 1                         
(b)  केवल 2
(c)  1 और 2 दोनों    
(d)  न तो 1 और न ही 2

उत्तर:(c) 

व्याख्या: पुन: संयोजित (Recombinant) वेक्टर वैक्सीन एक नवीन तकनीक है। इसमें वैक्सीन की प्रतिकृति में एक पूरी तरह से सक्षम वायरल वेक्टर आधार होता है। इसे इस तरह बनाया जाता है कि जिससे एक विदेशी ट्रांसजीन को एंटीजन में व्यक्त कर सकें। पुन: संयोजित वेक्टर वैक्सीन जीवित प्रतिकृति वायरस (Live Replicating Viruses) होते हैं जिन्हें एक रोगजनक (Pathogen) से प्राप्त अतिरिक्त जीन को ले जाने के लिये इंजीनियर किया जाता है और ये अतिरिक्त जीन प्रोटीन उत्पन्न करते हैं जिसके खिलाफ हम प्रतिरक्षा उत्पन्न करना चाहते हैं। अत: कथन (1) सही है।

  • रोगवाहक (Vector) जीवित जीव हैं जो मनुष्यों के बीच या जानवरों से मनुष्यों के बीच संक्रामक रोगजनकों को प्रसारित कर सकते हैं। अक्सर, एक बार जब एक वेक्टर संक्रामक हो जाता है, तो वे अपने शेष जीवन के लिये रोगजनक को प्रसारित करने में सक्षम होते हैं। वैक्सीन एक सुरक्षित जीवाणुओं और विषाणुओं का उपयोग करते हैं। इनके विशिष्ट भाग, जिसे प्रोटीन कहा जाता है, का उपयोग किया जाता है। संबंधित रोगजनक का इस प्रकार उपयोग किया जाता है कि यह रोग पैदा किये बिना प्रतिरक्षा प्रतिक्रिया को ट्रिगर कर सकें। अत: कथन (2) सही है। अत: विल्कप (c) सही है।

76. आनुवंशिक रोगों के संदर्भ में, निम्नलिखित कथनों पर विचार कीजिये:

  1. अंडों के अंत:पात्र (इन विट्रो) निषेचन से या तो पहले या बाद में सूत्रकणिका प्रतिस्थापन (माइटोकॉण्ड्रियल रिप्लेसमेंट) चिकित्सा द्वारा सूत्रकणिका रोगों (माइटोकॉण्ड्रियल डिजीज़) को माता-पिता से संतान में जाने से रोका जा सकता है।
  2. किसी संतान में सूत्रकणिका रोग आनुवंशिक रूप से पूर्णत: माता से जाता है न कि पिता से।

उपर्युक्त कथनों में से कौन-सा/से सही है/हैं?

(a) केवल 1                         
(b)  केवल 2
(c)  1 और 2 दोनों    
(d)  न तो 1 और न ही 2

उत्तर:(c)

व्याख्या: सूत्रकणिका प्रतिस्थापन चिकित्सा (Mitochondrial Replacement Therapy) में माता के खराब माइटोकॉण्ड्रिया को दाता (Doner) महिला के स्वस्थ माइटोकॉण्ड्रिया के द्वारा बदला जाता है। उसके बाद इनविट्रो फर्टिलाइजेशन (IVF) तकनीक के द्वारा अंडाणु (Ovum) और सहयोगी के शुक्राणु (Sperm) के साथ निषेचन से प्राप्त युग्मनज (Zygote) का प्रारंभिक भ्रूणीय विकास आंरम्भ कराया जाता है। इसके माध्यम से विकारयुक्त माइटोकॉण्ड्रिया डी.एन.ए को दुरुस्त किया जा सकता है तथा इससे उत्पन्न अन्य बीमारियों को रोका जा सकता है। अत: कथन (1) सही है। विकल्प (c) सही उत्तर होगा।

  • मनुष्य केवल अपनी माता से माइटोकॉण्ड्रिया प्राप्त करता है और माइटोकॉण्ड्रिया डी.एन.ए (Mt. DNA) से प्राप्त करते हैं। अत: कथन (2) सही है।

77. बॉलगार्ड-I और बॉलगार्ड--II प्रौद्योगिकियों का उल्लेख किसके संदर्भ में किया जाता है?

(a) फसली पादपों का क्लोनी प्रवर्धन
(b) आनुवंशिक रूप से रूपांतरित फसली पादपों का विकास
(c) पादप वृद्धिकर पदार्थों का उत्पादन
(d) जैव उर्वरकों का उत्पादन

उत्तर:(b)

व्याख्या: बॉलगार्ड-I बीटी कपास से संबंधित तकनीक है। यह भारत की पहली बायौटेक फसल तकनीक है जिसे वर्ष 2002 में भारत में व्यवसायीकरण के लिये अनुमोदित किया गया है।

  • बॉलगार्ड-I एकल जीन प्रौद्योगिकी (Cry-1Ac) है तो वहीं बॉलगार्ड-II  डबल जीन प्रौद्योगिकी (Cry-2Ac, Cry-1Ab) है। अत: विकल्प (b) सही है।

78. किसी प्रेशर कुकर में, जिस तापमान पर खाद्य पकाए जाते हैं, वह मुख्यत: निम्नलिखित में से किन पर निर्भर करता है?

  1. ढक्कन में स्थित छिद्र का क्षेत्रफल
  2. ज्वाला का तापमान
  3. ढक्कन का भार

नीदे दिये गए कूट का प्रयोग कर सही उत्तर चुनिये-

(a) केवल 1 और 2             
(b)  केवल 2 और 3
(c) केवल 1 और 3  
(d) 1, 2 और 3

उत्तर:(d) 

व्याख्या: प्रेशर कुकर में खाना इसलिये जल्दी पक जाता है क्योंकि अत्यधिक दाब पर क्वथनंाक बढ़ जाता है और ज़्यादा ऊष्मा देने पर ही क्वथन संभव हो पाता है। वस्तुत: ढक्कन में स्थित छिद्र का क्षेत्रफल कम होने पर दाब अधिक होगा तथा ज्वाला का तापमान एवं ढक्कन का भार अधिक होने पर क्वथनांक बढ़ने तथा अधिक दाब होने पर खाना जल्दी पकेगा। अत: विकल्प (d) सही है।

79. निम्नलिखित पर विचार कीजिये:

1. जीवाणु  
2. कवक  
3. विषाणु

उपर्युक्त में से किन्हें कृत्रिम/संश्लेषित माध्यम में संवर्धित किया जा सकता है?

(a) केवल 1 और 2           
(b)  केवल 2 और 3
(c) केवल 1 और 3           
(d) 1, 2 और 3

उत्तर:(a) 

व्याख्या: वायरस वे अकोशिकीय अतिसूक्ष्म जीव हैं जो जीवित कोशिका में ही पुनरूत्पादन कर सकते हैं। ये शरीर के बाहर मृत या सुसुप्तावस्था में होते हैं परंतु शरीर या जीवित माध्यम के संपर्क में आने पर जीवित हो जाते हैं। ये केवल जीवित कोशिका में ही वृद्धि कर सकते हैं अत: इन्हें कृत्रिम रूप में संवर्द्धित नहीं किया जा सकता।

  • बैक्टिीरिया कोशिकीय सूक्ष्मजीव होते हैं तथा कवक बहुकोशिकीय यूकैरियोटिक जीव हैं। बैक्टिीरिया एंव कवक दोनों को कृत्रिम रूप में सवंर्द्धित किया जा सकता है। अत: विकल्प (a) सही है।

80. निम्नलिखित कथनों पर विचार कीजिये:

  1. एडीनोवायरसों में एकल-तंतु डी.एन.ए. संजीन (जीनोम) होते हैं, जबकि रेट्रोवायरसों में द्वि-तंतु डी.एन.ए. संजीन (जीनोम) होते हैं।
  2. कभी-कभी सामान्य जुकाम एडीनोवायरस के कारण होता है, जबकि एड्स (ए.आई.डी.एस.) रेट्रोवायरस के कारण होता है।

उपर्युक्त कथनों में से कौन-सा/से सही है/हैं?

(a) केवल 1                         
(b)  केवल 2
(c)  1 और 2 दोनों    
(d)  न तो 1 और न ही 2

उत्तर:(b)

व्याख्या: एडीनोवायरस मध्यम आकार का एक न्यूक्लियोकैप्सिड (Nucleocapsid) और एक रैखिक द्वि-तंतु डी.एन.ए. संजीन (Linear Double-stranded DNA Genome) से बना आईकोसाहेड्रल (icosahedral) वायरस है। जबकि रेट्रोवायरस एकल-तंतु डी.एन.ए. संजीन आरएनए पशु वायरस (Single-Stranded RNA Animal Viruses) है जो प्रतिकृति के लिये द्वि-तंतु- डीएनए मध्यवर्ती (Double-stranded DNA Intermediate) को नियोजित करते हैं। अत: कथन (1) सही नहीं है।

  • एडीनोवायरस आम वायरस है जो कई तरह की बीमारियों का कारण बनते हैं। ये सामान्य जुकाम, बुखार, गले में खराश, ब्रोंकाइटिस (Bronchitis), निमोनिया, दस्त और गुलाबी आँख (Conjunctivitis) पैदा कर सकते हैं। एडीनोवायरस संक्रमण किसी भी उम्र में हो सकता है। कमज़ोर प्रतिरक्षा प्रणाली या मौजूदा श्वसन या हृदय रोग वाले लोगों में एडीनोवायरस संक्रमण से बीमार होने की संभावना अधिक होती है। एचआईवी को रेट्रोवायरस कहा जाता है क्योंकि यह बैक-टू-फ्रंट तरीके से काम करता है। अन्य वायरस के विपरीत, रेट्रोवायरस डीएनए की बजाय आरएनए का उपयोग करके अपनी आनुवंशिक जानकारी संगृहीत करते हैं, जिसका अर्थ है कि जब वे मानव कोशिका में प्रवेश करते हैं तो उन्हें स्वयं की नई प्रतियाँ बनाने के लिये डीएनए को बनाने की आवश्यकता होती है। अत: कथन (2) सही है। अत: विकल्प (b) सही है।

81. स्थायी कृषि (पर्माकल्चर), पारंपरिक रासायनिक कृषि से किस तरह भिन्न है?

  1. स्थायी कृषि एकधान्य कृषि पद्धति को हतोत्साहित करती है, किन्तु पारंपरिक रासायनिक कृषि में एकधान्य कृषि पद्धति की प्रधानता है।
  2. पारंपरिक रासायनिक कृषि के कारण मृदा की लवणता में वृद्धि हो सकती है, किन्तु इस तरह की परिघटना स्थायी कृषि में दृष्टिगोचर नहीं होती है।
  3. पारंपरिक रासायनिक कृषि अर्धशुष्क क्षेत्रों में आसानी से संभव है, किन्तु ऐसे क्षेत्रों में स्थायी कृषि इतनी आसानी से संभव नहीं है।
  4. मल्च बनाने (मल्ंिचग) की प्रथा स्थायी कृषि में काफी महत्त्वपूर्ण है, किन्तु पारंपरिक रासायनिक कृषि में ऐसी प्रथा आवश्यक नहीं है।

नीचे दिये गए कूट का प्रयोग कर सही उत्तर चुनिये-

(a)  1 और 3                    
(b)  1, 2 और 4
(c)  केवल 4                      
(d)  2 और 3

उत्तर:(b) 

व्याख्या: स्थायी कृषि (पर्माकल्चर) प्राकृतिक पारिस्थितिक तंत्र के अंतर्संबंधों और स्थिरता को प्रतिबिंबित करती है। पर्माकल्चर भूमि का सर्वोत्तम उपयोग करने का एक प्रयास है ताकि भविष्य में आने वाली पीढ़ियाँ उत्पादक तरीके से भूमि का उपयोग जारी रख सकें। पर्माकल्चर एकधान्य कृषि को हतोत्साहित करता है और खाद्यान्न, फलों और सब्ज़ियों की एक विस्तृत  शृंखला को उगाने और इस तरह खाद्य टोकरी का विस्तार करने की संभावना का मार्ग प्रशस्त करता है। पर्माकल्चर विधियों के अनुप्रयोग और पर्माकल्चर तकनीकों जैसे कि प्राकृतिक मल्च़िग, वर्षा जल संचयन, मिट्टी के गुणों में सुधार, मिट्टी में कार्बनिक पदार्थ की मात्रा बढ़ाने और मिट्टी की लवणता को कम करने में एक स्पष्ट भूमिका है। मल्च़िग, फसल की उपज में सुधार करने और पानी के उपयोग को अनुकूलित करने में मदद कर सकती है जो कि पर्माकल्चर का एक अनिवार्य घटक है। जल संरक्षण और क्षेत्र विशिष्ट फसलों पर केंद्रित होने के कारण यह शुष्क और अर्द्ध-शुष्क क्षेत्रों के लिये अधिक उपयुक्त है। अत: विकल्प (b) सही है।

82. ‘ताड़ तेल (पाम ऑयल)’ के संदर्भ में, निम्नलिखित कथनों पर विचार कीजिये:

  1. ताड़ तेल वृक्ष दक्षिण-पूर्व एशिया में प्राकृतिक रूप में पाया जाता है।
  2. ताड़ तेल लिपस्टिक और इत्र बनाने वाले कुछ उद्योगों के लिये कच्चा माल है।
  3. ताड़ तेल का उपयोग जैव डीज़ल के उत्पादन में किया जा सकता है।

उपर्युक्त कथनों में से कौन-से सही हैं?

(a) केवल 1 और 2  
(b)  केवल 2 और 3
(c) केवल 1 और 3  
(d) 1, 2 और 3

उत्तर:(b) 

व्याख्या: ताड़ तेल वृक्ष अफ्रीका में प्राकृतिक रूप से पाया जाता है। वैश्विक बाज़ार में बायोडीजल उत्पादन में और विविधता लाने के लिये पाम तेल को एक वैकल्पिक और आशाजनक फीडस्टॉक माना जाता है। यह एक वनस्पति तेल है, जिसका उपयोग लिपस्टिक व इत्र (Perfume) जैसे सौंदर्य प्रसाधनों (Cosmetics) में नमी प्रदायक (moisturising) और टेक्सचराइजिंग (Texturising) गुणों के लिये किया जाता है। अत: विकल्प (b) सही है।

83. सिंधु नदी प्रणाली के संदर्भ में, निम्नलिखित चार नदियों में से तीन नदियाँ इनमें से किसी एक नदी में मिलती हैं जो सीधे सिंधु नदी से मिलती है। निम्नलिखित में से वह नदी कौन-सी है, जो सिंधु नदी से सीधे मिलती है?

(a) चेनाब         
(b)  झेलम
(c) रावी           
(d)  सतलुज

उत्तर:(d) 

व्याख्या: सिंधु नदी का उद्गम तिब्बत में कैलाश पर्वत श्रेणी में ‘बोखार चू (Bokhar Chu) के निकट एक हिमनद से होता है, इसे तिब्बत में ‘सिंगी खंबान अथवा ‘शेर मुख कहते हैं। इस नदी के बाएँ और दाएँ दोनों तरफ से अनेक सहायक नदियाँ मिलती हैं, जैसे-श्योक, गिलगित,शिगार, काबुल, जास्कर, पंचनद (झेलम, चेनाब, रावी, व्यास, सतलुज)आदि। पंचनद नदियाँ आपस में मिलकर पाकिस्तान में ‘मिथनकोट के पास सिंधु नदी में मिल जाती हैं। ध्यातव्य है कि सतलुज नदी सिंधु नदी में सीधे मिलती है। अत: विकल्प (d) सही है।

84. भारत के संदर्भ में डीडवाना, कुचामन, सरगोल और खाटू किनके नाम हैं?

(a) हिमनद        
(b)  गरान (मैंग्रोव) क्षेत्र
(c) रामसर क्षेत्र     
(d)  लवण झील

उत्तर:(d) 

व्याख्या: हवाओं के प्रवाह एवं अपरदन से निर्मित झीलें ‘वायु द्वारा निर्मित झील की श्रेणी के अंतर्गत आती हैं। इन झीलों को प्लाया भी कहते हैं। ये मुख्यत: लवणीय झीलें होती हैं। राजस्थान की अधिकांश झीलें इसी श्रेणी की हैं,जैसे-सांभर, पंचभद्रा, लूणकरणसर, डीडवाना, कुचामन, सरगोल, खाटू आदि। अत: विकल्प (d) सही है।

85. निम्नलिखित नदियों पर विचार कीजिये:

1. ब्राह्मणी        
2. नागावली
3. सुवर्णरेखा       
4. वंशधारा

उपर्युक्त में से कौन-सी नदियाँ पूर्वी घाट से निकलती हैं?

(a)  1 और 2                        
(b) 2 और 4
(c)  3 और 4       
(d) 1 और 3

उत्तर:(b) 

व्याख्या: पूर्वी घाट की असंबद्ध पहाड़ी  शृंखलाएँ ओडिशा, तेलंगाना, आंध्र प्रदेश, कर्नाटक और तमिलनाडु में फैली हुई हैं जो कि अद्वितीय पारिस्थितिकी तंत्र का उदाहरण हैं। नागावली नदी का उद्गम ओडिशा के कालाहांडी ज़िले में 1,300 मीटर की ऊँचाई पर स्थित लखबहल के पास पूर्वी घाट के पूर्वी ढलानों से होता है। वंशधारा नदी ओडिशा के कालाहांडी और रायगढ़ ज़िले की सीमा पर पूर्वी घाट से निकलती है। वहीं ब्राह्मणी नदी का उद्गम राउरकेला (ओडिशा) के निकट दक्षिणी कोयल एवं शंख नदियों के मिलने से तथा सुवर्णरेखा का उद्गम छोटानागपुर के पठार से होता है। अत: विकल्प (b) सही है।

86. निम्नलिखित कथनों पर विचार कीजिये-

  1. वैश्विक सागर आयोग (ग्लोबल ओशन कमीशन) अंतर्राष्ट्रीय जल-क्षेत्र में समुद्र-संस्तरीय (सीबेड) खोज और खनन के लिये लाइसेंस प्रदान करता है।
  2. भारत ने अंतर्राष्ट्रीय जल-क्षेत्र में समुद्र-संस्तरीय खनिज की खोज के लिये लाइसेंस प्राप्त किया है।
  3. ‘दुर्लभ मृदा खनिज (रेयर अर्थ मिनरल)’ अंतर्राष्ट्रीय जल-क्षेत्र में समुद्र अधस्तल पर उपलब्ध है।

उपर्युक्त कथनों में से कौन-से सही हैं?

(a) केवल 1 और 2             
(b)  केवल 2 और 3
(c) केवल 1 और 3  
(d) 1, 2 और 3

उत्तर:(b)

व्याख्या: वैश्विक सागर आयोग 2013 से 2016 के बीच समुद्री तंत्र के क्षरण को संबोधित करने,इसके लिये जागरूकता बढ़ाने और इसकी उत्पादकता को बहाल करने में मदद करने के लिये एक अंतर्राष्ट्रीय पहल थी। जबकि समुद्र संस्तरीय (सीबेड) खोज और खनन के लिये लाइसेंस प्रदान करने का कार्य अंतर्राष्ट्रीय समुद्री प्राधिकरण (International Seabed Authority-ISA) द्वारा किया जाता है। यह संयुक्त राष्ट्र संघ का एक निकाय है, जिसे अंतर्राष्ट्रीय जल के अंतर्गत महासागरों में पाए जाने वाले निर्जीव संसाधनों के संबंध में अन्वेषण व दोहन आदि कार्यों को विनियमित करने के लिये स्थापित किया गया है।  

  • विदित हो कि केंद्रीय भारतीय महासागरीय बेसिन (Central Indian Ocean Basin- CIOB) के समुद्र तट में पॉलीमेटैलिक ग्रंथियों (polymetallic nodules) का पता लगाने संबंधी भारत के विशेषाधिकार को वर्ष 2017 में पाँच साल के लिये बढ़ा दिया गया है। अंतर्राष्ट्रीय जलक्षेत्र में दुर्लभ मृदा खनिज समुद्र अधस्थल पर उपलब्ध होते हैं। दुर्लभ मृदा खनिज में प्रमुख धातु घटकों के रूप में एक या अधिक दुर्लभ मृदा तत्त्व होते हैं। ये पृथ्वी की ऊपरी सतह (क्रस्ट) में पाए जाते हैं। दुर्लभ मृदा तत्त्व अपने नाम के विपरीत पृथ्वी पर प्रचुर मात्रा में पाए जाते हैं लेकिन इनके प्रसंस्करण की प्रक्रिया बहुत ही जटिल है। अत: विकल्प (b) सही है।

87. निम्नलिखित में से कौन-सी फसल, न्यूनतम जल-दक्ष (लीस्ट वाटर-एफिशिएंट) फसल है?

(a) गन्ना         
(b)  सूरजमुखी
(c) बाजरा         
(d)  अरहर (रेड ग्राम)

उत्तर:(a)

व्याख्या: गन्ना एक नकदी फसल है,जिसके उत्पादन के लिये 75&150 सेमी. वार्षिक वर्षा की आवश्यकता होती है। एक किलो ग्राम गन्ने के उत्पादन में करीब 210 लीटर पानी खर्च होता है। वहीं सूरजमुखी के प्रति किलो ग्राम उत्पादन में 7-9 लीटर पानी की खपत होती है। बाजरे की बुवाई शुष्क क्षेत्रों में न्यूनतम सिंचाई के साथ की जाती है। अरहर (लाल चना) एक वर्षा आधारित फसल है जो निश्चित वर्षा वाले क्षेत्रों में उगाई जाती है। इस फसल के लिये 35 से 40 सेमी. पानी के साथ-साथ नमी की भी आवश्यकता होती है। आमतौर पर इसे किसी सिंचाई की आवश्यकता नहीं होती है। इस प्रकार उपर्युक्त विकल्पों में गन्ना ही वह फसल है जो न्यूनतम जल-दक्ष (Least Water-efficient) है। अत: विकल्प (a) सही है।

88. निम्नलिखित कथनों पर विचार कीजिये-

  1. उष्णकटिबंधीय क्षेत्र में, व्यापारिक पवन के प्रभाव के कारण पूर्वी खंडों की तुलना में महासागरों के पश्चिमी खंड अधिक उष्ण होते हैं।
  2. शीतोष्ण क्षेत्र में, पश्चिमी पवन पश्चिमी खंडों की तुलना में महासागरों के पूर्वी खंडों को अधिक उष्ण बनाती है।

उपर्युक्त कथनों में से कौन-सा/से सही है/हैं?

(a) केवल 1                         
(b)  केवल 2
(c)  1 और 2 दोनों    
(d)  न तो 1 और न ही 2

उत्तर:(c)

व्याख्या: उपोष्ण उच्च वायुदाब कटिबंधों में भूमध्यरेखीय निम्न वायुदाब कटिबंधों की ओर चलने वाली पवनों को ‘सन्मार्गी/व्यापारिक पवनें कहते हैं। ये पवनें महासागरों के पूर्वी खंड से गर्म और हल्के जल को आगे की ओर लेकर जाती हैं, जिससे महासागर के निचले क्षेत्र का ठंडा जल ऊपर आ जाता है और इस प्रकार इन पवनों से महासागरों के पूर्वी खंड ठंडे और पश्चिमी खंड अपेक्षाकृत अधिक गर्म हो जाते हैं। इसी प्रकार शीतोष्ण क्षेत्र में उपोष्ण उच्च वायुदाब कटिबंध से उपध्रुवीय निम्न वायुदाब कटिबंधों की ओर चलने वाली पश्चिमी पवनें (Westerlies) महासागरों के पश्चिमी खंड के गर्म और हल्के जल को आगे की ओर हटाती हैं, जिससे पूर्वी खंड गर्म हो जाते हैं। अत: विकल्प (c) सही है।

89. जलवायु-अनुकूल कृषि (क्लाइमेट-स्मार्ट एग्रीकल्चर) के लिये भारत की तैयारी के संदर्भ में, निम्नलिखित कथनों पर विचार कीजिये-

  1. भारत में ‘जलवायु-स्मार्ट ग्राम (क्लाइमेट-स्मार्ट विलेज)’ दृष्टिकोण, अंतर्राष्ट्रीय अनुसंधान कार्यक्रम-जलवायु परिवर्तन, कृषि एवं खाद्य सुरक्षा (सी.सी.ए.एफ.एस.) द्वारा संचालित परियोजना का एक भाग है।
  2. सी.सी.ए.एफ.एस. परियोजना, अंतर्राष्ट्रीय कृषि अनुसंधान हेतु परामर्शदात्री समूह (सी.जी.आई.ए.आर.) के अधीन संचालित किया जाता है, जिसका मुख्यालय प्राँस में है।
  3. भारत में स्थित अंतर्राष्ट्रीय अर्धशुष्क उष्णकटिबंधीय फसल अनुसंधान संस्थान (आई.सी.आर.आई.एस.ए.टी.), सी.जी.आई.ए.आर. के अनुसंधान केंद्रों में से एक है।

उपर्युक्त कथनों में से कौन-से सही हैं?

(a) केवल 1 और 2             
(b)  केवल 2 और 3
(c) केवल 1 और 3  
(d) 1, 2 और 3

उत्तर:(d)

व्याख्या: ‘जलवायु स्मार्ट कृषि (Climate Smart Agriculture-CSA) एक दृष्टिकोण है, जो बदलती हुई जलवायु में खाद्य सुरक्षा सुनिश्चित करने हेतु कृषि प्रणालियों को परिवर्तित और पुनर्जीवित करने के लिये आवश्यक क्रियाओं को निर्देशित करने में मदद करता है। देश में जलवायु-स्मार्ट कृषि विकसित करने की ठोस पहल की गई है और इसके लिये राष्ट्रीय स्तर की परियोजना भी लागू की गई है। यह परियोजना खाद्य सुरक्षा और जलवायु परिवर्तन की परस्पर चुनौतियों का सामना करने के लिये बनाई गई है। इसके लिये ‘जलवायु स्मार्ट ग्राम तैयार किये गए है,जिसमें उचित फसल, किस्मों, प्राकृतिक संसाधन प्रबंधन, कृषि यंत्रीकरण एवं कस्टम हायरिंग केंद्रों पर ध्यान केंद्रित किया गया है। यह दृष्टिकोण ‘अंतर्राष्ट्रीय अनुसंधान कार्यक्रम-जलवायु परिवर्तन, कृषि एवं खाद्य सुरक्षा द्वारा संचालित परियोजना का एक भाग है जो एक परामर्शदात्री समूह के अधीन संचालित किया जाता है। इसका मुख्यालय फ्राँस में है। अर्द्ध-शुष्क उष्णकटिबंधीय अंतर्राष्ट्रीय फसल अनुसंधान संस्थान (ICRISAT) एक अंतर्राष्ट्रीय संगठन है, जो ग्रामीण विकास के लिये कृषि अनुसंधान कार्य करता है। इसका मुख्यालय पाटनचेरू (हैदराबाद, तेलंगाना) में स्थित है। अत: विकल्प (d) सही है।

90. ‘‘पत्ती-कूड़ा (लीप़ लिटर) किसी अन्य जीवोम (बायोम) की तुलना में तेज़ी से विघटित होता है और इसके परिणामस्वरूप मिट्टी की सतह प्राय: अनावृत होती है। पेड़ों के अतिरिक्त, वन में विविध प्रकार के पौधे होते हैं जो आरोहण के द्वारा या अधिपादप (एपिफाइट)के रूप में पनपकर पेड़ों के शीर्ष तक पहुँचकर प्रतिस्थ होते हैं और पेड़ों की ऊपरी शाखाओं में जड़ें जमाते हैं।’’ यह किसका सबसे अधिक सटीक विवरण है?

(a) शंकुधारी वन    
(b)  शुष्क पर्णपाती वन
(c) गरान (मैंग्रोव) वन
(d)  उष्णकटिबंधीय वर्षावन

उत्तर:(d) 

व्याख्या: उष्णकटिबंधीय सदाबहार वन को उष्णकटिबंधीय वर्षावन भी कहते हैं। ये वन सघन और पत्तों वाले होते हैं, जहाँ भूमि के नज़दीक झाड़ियाँ और बेलें होती हैं, इनके ऊपर छोटे कद वाले पेड़ और सबसे ऊपर लंबे पेड़ होते हैं। इन वनों में वृक्षों की लंबाई 60 मी. या उससे भी अधिक हो सकती है। चूँकि, इन पेड़ों के पत्ते झड़ने, फूल आने और फल लगने का समय अलग-अलग होता है, इसलिये ये वर्ष भर हरे-भरे दिखाई देते हैं। किसी अन्य जीवोम (बायोम) की तुलना में यहाँ पत्ती-कूड़ा तेज़ी से विघटित होता है, जिससे मिट्टी की सतह प्राय: अनावृत्त होती है। इसमें प्रमुख रूप से पाई जाने वाली वृक्ष प्रजातियाँ रोजवुड, महोगनी, ऐनी और ऐबनी हैं। अत: विकल्प (d) सही है।

91. जल किसी अन्य द्रव की अपेक्षा अधिक पदार्थों को घोल सकता है, क्योंकि

(a) इसकी प्रकृति द्विध्रुवीय है
(b) यह ऊष्मा का सुचालक है
(c) इसकी विशिष्ट ऊष्मा का मान उच्च होता है
(d) यह हाइड्रोजन का एक ऑक्साइड है

उत्तर:(a) 

व्याख्या: जल को सार्वभौमिक विलायक कहा जाता है क्योंकि यह किसी भी अन्य द्रव की तुलना में अधिक पदार्थों को घोलने में सक्षम है। जल अपनी द्विध्रुवीय प्रकृति के कारण किसी भी अन्य यौगिक की तुलना में अधिक पदार्थों को घोल सकता है। पानी के अणु अपनी विशिष्ट संरचना के कारण, एक तरफ धनात्मक आवेश वाले हाइड्रोजन और दूसरी ओर ऋणात्मक आवेश वाली ऑक्सीजन, अन्य अणुओं को आसानी से आकर्षित करने में सक्षम होते हैं। अत: विकल्प (a) सही है।

92. सड़क प्रकाश व्यवस्था के संदर्भ में, सोडियम बत्तियाँ, एल.ई.डी. बत्तियों से किस तरह भिन्न हैं?

  1. सोडियम बत्तियाँ प्रकाश को 360 डिग्री में उत्पन्न करती हैं, किंतु एल.ई.डी. बत्तियों में ऐसा नहीं होता है।
  2. सड़क की बत्तियों के रूप में, एल.ई.डी. बत्तियों की तुलना में सोडियम बत्तियों की उपयोगिता अवधि अधिक होती है।
  3. सोडियम बत्ती के दृश्य प्रकाश का स्पेक्ट्रम लगभग एकवर्णी होता है, जबकि एल.ई.डी. बत्तियाँ सड़क प्रकाश व्यवस्था में सार्थक वर्ण सुविधाएँ (कलर एडवैंटेज) प्रदान करती हैं।

नीचे दिये गए कूट का प्रयोग कर सही उत्तर चुनिये-

(a) केवल 3                         
(b)  केवल 2
(c) केवल 1 और 3  
(d) 1, 2 और 3

उत्तर:(c) 

व्याख्या: हाई प्रेशर सोडियम बत्तियाँ (Sodium Lamps) सर्वदिशिक (Omnidirectional) होती हैं, जो 360° में प्रकाश उत्पन्न करती हैं। इसमें प्रकाश का अधिक अपव्यय होता है जो इसे कम कुशल बनाता है। वहीं एल.ई.डी. बत्तियाँ (LED Lamps) प्रकाश दक्षता को बनाए रखने और लक्षित क्षेत्रों पर प्रकाश उत्पन्न करने के लिये 180° में प्रकाश उत्पन्न करती हैं। अत: कथन (1) सही है।

  • सड़क की बत्तियों के रूप में सोडियम बत्तियों की तुलना में एल.ई.डी.बत्तियों की उपयोगिता अवधि अधिक होती है। एल.ई.डी.बत्तियों की उपयोगिता अवधि सोडियम बत्तियों की तुलना में लगभग 4 गुना अधिक होती है। अत: कथन (2) सही नहीं है।
  • सोडियम बत्तियाँ एक बहुत ही संकीर्ण आवृत्ति 589 और 589.56 नैनोमीटर के दो तरंग दैर्ध्य का उत्सर्जन करती हैं। इनके दृश्य प्रकाश का स्पेक्ट्रम एकवर्णी (Monochromatic)  होता है जो पीले रंग का उत्सर्जन करता है। जबकि एल.ई.डी. बत्तियों में बहुत ही विस्तृत आवृत्ति का तरंग दैर्ध्य उपलब्ध होता है। अत: कथन 3 सही है। अत: विकल्प (c) सही है।

93. 'ACE2' पद का उल्लेख किस संदर्भ में किया जाता है?

(a) आनुवंशिक रूप से रूपांतरित पादपों में पुन:स्थापित (इंट्रोड्यूस्ड) जीन
(b) भारत के निजी उपग्रह संचालन प्रणाली का विकास
(c) वन्य प्राणियों पर निगाह रखने के लिये रेडियो कॉलर
(d) विषाणुजनित रोगों का प्रसार

उत्तर:(d)

व्याख्या: हाल के शोधों के अनुसार मानव शरीर में ACE2 [एंजियोटेंसिन-परिवर्तित एंजाइम-2 (Angiotensin-Converting Enzyme 2)] वह प्रमुख कारक है जो COVID-19 को मानव कोशिकाओं को संक्रमित करने में सक्षम बनाता है। दरअसल मानव कोशिकाओं की सतह पर ACE2 नामक एक एंजाइम होता है, जो रिसेप्टर के रूप में कार्य करता है जो SARS-CoV2 को अपना हमला शुरू करने के लिये सक्षम करता है। वायरस का स्पाइक प्रोटीन रिसेप्टर से जुड़ जाता है, फिर कोशिका की सतह के साथ फ्यूज हो जाता है, और अपनी आनुवंशिक सामग्री (SARS-CoV2 के मामले में RNA) को कोशिका में छोड़ देता है। इस प्रकार ACE2 विषाणुजनित रोगों के प्रसार से संबंधित है। अत: विकल्प (d) सही है।

94. बिस्फिनॉल A (BPA), जो चिंता का कारण है, निम्नलिखित में से किस प्रकार के प्लास्टिक के उत्पादन में एक संरचनात्मक/मुख्य घटक है?

(a) निम्न घनत्व वाले पॉलियेथिलीन    
(b) पॉलिकार्बोनेट
(c) पॉलियेथिलीन टेरेफ्थेलेट 
(d) पॉलिविनाइल क्लोराइड

उत्तर:(b)

व्याख्या: बिस्फिनॉल A (BPA) पॉलिकार्बोनेट प्लास्टिक बनाने के लिये इस्तेमाल किया जाने वाला एक रसायन है। पॉलिकार्बोनेट प्लास्टिक का उपयोग कठोर प्लास्टिक की वस्तुओं, जैसे कि पुन: उपयोग करने योग्य पानी की बोतलें, बच्चों के दूध की बोतलें, खाद्य कंटेनर, टेबलवेयर और अन्य भंडारण योग्य कंटेनर बनाने के लिये किया जाता है। BPA एक सिंथेटिक ऑर्गेनिक कंपाउंड (Synthetic Organic Compound) है। BPA एपॉक्सी रेजिन में भी पाया जाता है, जो कुछ धातु-आधारित खाद्य और पेय के डिब्बे के अंदर एक सुरक्षात्मक अस्तर के रूप में कार्य करता है। पॉलिकार्बोनेट की बोतलों से तरल में BPA का रिसाव तरल या बोतल के तापमान पर अधिक निर्भर होता है। दरअसल BPA का लोगों पर व्यापक प्रभाव पड़ता है जो एक चिंता का कारण है। अत: विकल्प (b) सही है।

95. निम्नलिखित में से किसमें ‘ट्राइक्लोसन’ के विद्यमान होने की सर्वाधिक संभावना है, जिसके लंबे समय तक उच्च स्तर के प्रभावन में रहने को हानिकारक माना जाता है?

(a) खाद्य परिरक्षक 
(b)  फल पकाने वाले पदार्थ
(c) पुन:प्रयुक्त प्लास्टिक के पात्र        
(d) प्रसाधन सामग्री

उत्तर:(d)

व्याख्या: ट्राइक्लोसन कई उपभोक्ता उत्पादों में मिलाया जाने वाला एक घटक है जिसका उद्देश्य जीवाणु संदूषण को कम करना या रोकना है। इसे कुछ जीवाणुरोधी साबुन और बॉडी वॉश, टूथपेस्ट और कुछ सौंदर्य प्रसाधनों में मिलाया जाता है। हाल के एक अध्ययन में यह पाया गया है कि ट्राइक्लोसन न्यूरोटॉक्सिक प्रभाव पैदा कर सकता है और न्यूरॉन्स को नुकसान पहुँचा सकता है। अमेरिका के फूड एंड ड्रग एडमिनिस्ट्रेशन ने इसके उपयोग पर आंशिक प्रतिबंध लगा दिया है। अत: विकल्प (d) सही है।

96. खगोलीय दूरियाँ प्रकाश-वर्ष में मापे जाने का कारण निम्नलिखित में से कौन-सा है?

(a) तारकीय पिंडों के बीच की दूरियाँ परिवर्तित नहीं होती हैं।
(b) तारकीय पिंडों का गुरुत्व परिवर्तित नहीं होता है।
(c) प्रकाश सदैव सीधी रेखा में यात्रा करता है।
(d) प्रकाश की गति (स्पीड) सदैव एकसमान होती है।

उत्तर:(d) 

व्याख्या: एक प्रकाश वर्ष दूरी की माप है। एक प्रकाश वर्ष वह दूरी है जो प्रकाश की किरण एक पृथ्वी वर्ष में तय करती है। प्रकाश लगभग 3,00,000 किमी प्रति सेकंड की गति से यात्रा करता है और यह गति पूरे ब्रह्मांड में हमेशा एक समान रहती है। एक वर्ष में प्रकाश द्वारा तय दूरी 9.4611012 किलोमीटर है। आइंस्टीन के अनुसार इस ब्रह्मांड में केवल एक चीज़ निरपेक्ष है, वह है प्रकाश की गति, बाकी सब कुछ सापेक्ष है। अत: विकल्प (d) सही है।

97. हमने ब्रिटिश मॉडल पर आधारित संसदीय लोकतंत्र को अपनाया है, किंतु हमारा मॉडल उस मॉडल से किस प्रकार भिन्न है?

  1. जहाँ तक विधि-निर्माण का संबंध है, ब्रिटिश संसद सर्वोपरि अथवा संप्रभु है, किंतु भारत में संसद की विधि-निर्माण की शक्ति परिसीमित है।
  2. भारत में, संसद के किसी अधिनियम के संशोधन की संवैधानिकता से संबंधित मामले उच्चतम न्यायालय द्वारा संविधान पीठ को भेजे जाते हैं।

नीचे दिये गए कूट का प्रयोग कर सही उत्तर चुनिये-

(a) केवल 1       
(b)  केवल 2
(c)  1 और 2 दोनों   
(d)  न तो 1 और न ही 2

उत्तर:(c)

व्याख्या: विधि निर्माण के संदर्भ में ब्रिटिश संसद संप्रभु है, जबकि भारत में संसदीय संप्रभुता के स्थान पर संविधान की सर्वोच्चता को स्वीकार किया गया है, अत: संसद की विधि निर्माण की शक्ति संविधान के प्रावधानों के द्वारा परिसीमित की जाती है। अत: कथन 1 सही है।

  • संविधान के अनुच्छेद-145 के अंतर्गत सर्वोच्च न्यायालय को प्रदत्त शक्तियों के आधार पर संसद द्वारा पारित किसी अधिनियम में संशोधन की संवैधानिकता जाँचने, अर्थात् उसमें निहित विधि के सारवान प्रश्न को संबोधित करने के लिये, न्यायालय की संविधान पीठ को भेजा जा सकता है। अत: कथन 2 सही है। विकल्प (c) सही उत्तर होगा।

98. संघ सरकार के संदर्भ में, निम्नलिखित कथनों पर विचार कीजिये-

  1. एन. गोपालास्वामी आयंगर समिति ने सुझाव दिया था कि किसी मंत्री और किसी सचिव को प्रशासनिक सुधार करने और उसे बढ़ावा देने के लिये पूर्णत: नामित किया जाना चाहिये।
  2. प्रशासनिक सुधार आयोग, 1966 की संस्तुति के आधार पर वर्ष 1970 में कार्मिक विभाग का गठन किया गया और इसे प्रधानमंत्री के प्रभार के अधीन रखा गया।

उपर्युक्त कथनों में से कौन-सा/से सही है/हैं?

(a) केवल 1       
(b)  केवल 2
(c)  1 और 2 दोनों   
(d)  न तो 1 और न ही 2

उत्तर:(c) 

व्याख्या: प्रशासनिक सुधारों के लिये एक मंत्री व एक सचिव को नामित किये जाने की सिफारिश ‘एन. गोपालास्वामी आयंगर समिति द्वारा दी गई थी। अत: कथन 1 सही है।

  • 1970 में ‘प्रशासनिक सुधार आयोग, 1966’ की अनुशंसा पर कैबिनेट सचिवालय के प्रभार के अधीन कार्मिक विभाग का गठन किया गया था। 1985 में इसका प्रभार प्रधानमंत्री कार्यालय को हस्तांतरित कर दिया गया था। उल्लेखनीय है कि कैबिनेट सचिवालय प्रभावी रूप से प्रधानमंत्री के प्रभार के अधीन है। अत: कथन 2 सही है। विकल्प (c) सही उत्तर होगा।

99. भारत के संविधान के किस अनुच्छेद के अंतर्गत ‘निजता का अधिकार’ संरक्षित है?

(a) अनुच्छेद-15                 
(b)  अनुच्छेद-19
(c) अनुच्छेद-21                 
(d)  अनुच्छेद-29

उत्तर:(c)

व्याख्या: 24 अगस्त, 2017 को सर्वोच्च न्यायालय की 9 न्यायाधीशों वाली संवैधानिक पीठ ने ‘जस्टिस के.एस. पुटेास्वामी बनाम भारत संघ मामले’ में ऐतिहासिक निर्णय देते हुए ‘निजता के अधिकार’ को मौलिक अधिकार के रूप में मान्यता दी। न्यायालय ने कहा कि निजता का अधिकार अनुच्छेद 21 के तहत प्रदत्त प्राण और दैहिक स्वतंत्रता के अधिकार के अंतर्भूत (Intrinsic) भाग के रूप में और संविधान के भाग 3 द्वारा प्रत्याभूत (Guaranteed) स्वतंत्रताओं के एक हिस्से के रूप में संरक्षित है। अत: विकल्प (c) सही उत्तर होगा।

100. निम्नलिखित कथनों पर विचार कीजिये-

  1. भारत में ऐसा कोई कानून नहीं है जो प्रत्याशियों को किसी एक लोकसभा चुनाव में तीन निर्वाचन-क्षेत्रों से लड़ने से रोकता है।
  2. 1991 में लोकसभा चुनाव में श्री देवी लाल ने तीन लोकसभा निर्वाचन-क्षेत्रों से चुनाव लड़ा था।
  3. वर्तमान नियमों के अनुसार, यदि कोई प्रत्याशी किसी एक लोकसभा चुनाव में कई निवार्चन-क्षेत्रों से चुनाव लड़ता है, तो उसकी पार्टी को उन निर्वाचन-क्षेत्रों के उप-चुनावों का खर्च उठाना चाहिये, जिन्हें उसने खाली किया है बशर्ते वह सभी निर्वाचन-क्षेत्रों से विजयी हुआ हो।

उपर्युक्त कथनों में से कौन-सा/से सही है/हैं?

(a) केवल 1       
(b)  केवल 2
(c) 1 और 3       
(d) 2 और 3

उत्तर: (b)

व्याख्या:  जन प्रतिनिधित्व अधिनियम, 1951 की धारा 33 (7) के प्रावधानों के अनुसार कोई व्यक्ति 2 से अधिक लोकसभा क्षेत्रों में प्रत्याशी नहीं बन सकता है। अत: कथन 1 सही नहीं है।

  • निर्वाचन आयोग की वेबसाइट पर उपलब्ध चुनाव परिणाम डाटा के अनुसार, देवीलाल 1991 के चुनाव में केवल हरियाणा के रोहतक लोकसभा क्षेत्र से चुनाव लड़े थे। हालाँकि 1989 में वे 3 सीटों (रोहतक, सीकर व फिरोजपुर) से चुनाव लड़े थे। अत: कथन 2 सही है।
  • यदि कोई प्रत्याशी एकाधिक निर्वाचन क्षेत्रों से लोकसभा चुनाव लड़कर जीत जाता है और बाद में किसी जीते हुए निर्वाचन क्षेत्र को खाली करता है, तो उसके उप-चुनावों का खर्चा सामान्य स्थिति की तरह केंद्र सरकार ही उठाएगी। अत: कथन 3 सही नहीं है।
close
एसएमएस अलर्ट
Share Page
images-2
images-2